Download as pdf or txt
Download as pdf or txt
You are on page 1of 44

4

CHAPTER

DETERMINANTS
Chapter Objectives
This chapter will help you understand :
¾¾ Determinants : Introduction, Types of determinants, Properties of determinants, Area of triangles, Minors and co-factors,
Adjoint and inverse of a matrix and Application of determinants and matrices.

Quick Review
™™ The Wronskian of solutions of a linear ODE is a
determinant. It plays a central role in spectral theory
(Hill’s equation with periodic co-efficients), and TIPS…
therefore in stability analysis of travelling waves in Along with numerical, some theories and concepts
PDEs. are also important in this chapter.
™™ Perron’s eigen value of an irreducible non-negative
matrix is a very nice use of the multi-linearity of the Properties of determinants are very important,
determinant. especially properties number 6 mentioned in
™™ The nth root of the determinant is a concave function NCERT.
over the n × n Hermitian positive definite matrices.
This is at the basis of many developments in modern
In expansion of determinants, decide well before
analysis, via the Brunn-Minkowski inequality. solving about one style expansion by row or column.
™™ In control theory, the Routh-Hurwitz algorithm,
which checks whether a system is stable or not, is
based on the calculation of determinants. TRICKS…
™™ As mentioned by J.M., Slater determinants are used You could always do the calculation twice, once
in quantum chemistry. with the top row as a starting point and one (say)
with the bottom row.
Know the Links If the matrix is structured so that a certain row or
☞☞
www.sosmath.com/matrix/determ0/determ0.html column has a lot of zeros in it then you must be
☞☞
https://www.math.drexel.edu/~jwd25/LM_SPRING_07/ sure to take advantage of this.
lectures/lecture4B.html You might consider Pivotal Condensation. Pivotal
☞☞
https://www.ironsidegroup.com/.../determinants-the- condensation can be extremely tedious; it may-or-
answer-to-a-framework-manager-... may-not be time-effective in solving problems.

 Multiple Choice Questions ⇒ 2x 2 − 40 = 18 + 14 (1 mark each)


⇒ 2x = 32 + 40
2

2x 5 6 −2 72
Q. 1. If = , then the value of x is ⇒ x2 = = 36
8 x 7 3 2
(a) 3 (b) ±3 ∴ x = ±6
(c) ±6 (d) 6
a−b b+ c a
 [NCERT Exemp. Ex. 4.3, Q. 24, Page 80]
Ans. Correct option : (c) Q. 2. The value of determinant b − a c + a b is
Explanation : Given that c−a a+ b c
2x 5 6 −2 (a) a3 + b3 + c 3 (b) 3bc
∴ = ,
8 x 7 3 (c) a3 + b3 + c 3 − 3 abc (d) None of these
 [NCERT Exemp. Ex. 4.3, Q. 25, Page 80]
⇒ 2x 2 − 40 = 18 + 14
Ans. Correct option : (d)
⇒ 2x 2 = 32 + 40
Explanation : We have
72
⇒ x2 = = 36
2
∴ x = ±6
bc − ac c − a ab − a c( b − a) c − a a(b − a)
b b−c c
= ( b − a )2 a a − b b
c c−a a
108 | OSWAAL NCERT SOLUTIONS – Textbook + Exemplar – MATHEMATICS : Class-XII [On taking ( b − a) common
from C1 and C3 each]
a−b b+c a a+c b+c+a a b−c b−c c
b−a c+a b = b+c c+a+b b = (b − a) a − b a − b b
2

c−a a+b c c+b a+b+c c c−a c−a a

[ C1 → C1 + C2 and C2 → C2 + C3 ] [ C1 → C1 − C3 ]
=0
a+c 1 a [Since, two columns C1 and C2 are identical, so the
= (a + b + c) b + c 1 b value of determinant is zero.]
c+b 1 c Q. 5. The number of distinct real roots of
[Taking ( a + b + c ) common from C2 ] sin x cos x cos x
π π
cos x sin x cos x = 0 in the interval − ≤ x ≤
a−b 0 a−c 4 4
cos x cos x sin x
= (a + b + c) 0 0 b−c
c+b 1 c is
(a) 0 (b) 2
[ R2 → R2 − R3 and R1 → R1 − R3 ] (c) 1 (d) 3
= ( a + b + c ) – [( b − c ) ( a − b )]  [NCERT Exemp. Ex. 4.3, Q. 28, Page 81]
Ans. Correct option : (c)
[Expanding along R2 ] Explanation : We have,
= ( a + b + c )( b − c )( a − b ) sin x cos x cos x
Q. 3. The area of a triangle with vertices (–3, 0), (3, 0) cos x sin x cos x = 0
and (0, k) is 9 sq. units. Then, the value of k will be
(a) 9 (b) 3 cos x cos x sin x
(c) –9 (d) 6 Applying C1→C1+C2+C3
 [NCERT Exemp. Ex. 4.3, Q. 26, Page 80] 2 cos x + sin x cos x cos x
Ans. Correct option : (b)
2 cos x + cos x sin x cos x = 0
Explanation : We know that, area of a triangle with
vertices (x1, y1), (x2, y2) and (x3, y3) is given by 2 cos x + cos x cos x sin x
x1 y1 1 On taking ( 2 cos x + sin x ) common from
1
∆ = x2 y2 1 the C1 , we get
2
x3 y3 1 1 cos x cos x
−3 0 1 ⇒ ( 2 cos x + sin x ) 1 sin x cos x
1
∴ ∆= 3 0 1 1 cos x sin x )
2
0 k 1 [ R2 → R2 − R1 and R3 → R3 − R1 ]
[Expanding along R1 ] 1 cos x cos x
1 ⇒ ( 2 cos x + sin x ) 0 sin x − cos x 0 =0
9 = [ −3( − k ) − 0 + 1(3k )]
2 0 0 (sin x − cos x )
⇒ 18 = 3k + 3k = 6k Expanding along C1,
18 ( 2 cos x + sin x )[1.(sin x − cos x )2 ] = 0
∴ k= =3
6 ⇒ ( 2 cos x + sin x )(sin x − cos x )2 = 0
b2 − ab b − c bc − ac Either 2 cos x = − sin x
Q. 4. The determinant ab − a2 a − b b2 − ab is equal 1
⇒ cos x = − sin x
2
bc − ac c − a ab − a2 ⇒ tan x = −2
to
But here for, we get − 1 ≤ tan x ≤ 1. So,
(a) abc ( b − c )( c − a )( a − b )
no solution is possible and for
(b) ( b − c )( c − a )( a − b )
(sin x − cos x )2 = 0, sin x = cos x
(c) (a + b + c ) ( b − c )( c − a )( a − b ) p
(d) None of these ⇒ tan x = 1 = tan ...(i)
4
 [NCERT Exemp. Ex. 4.3, Q. 27, Page 80] π
∴ x=
Ans. Correct option : (d) 4
Explanation : We have Thus, one distinct real root exists.
b 2 − ab b − c bc − ac b(b − a) b − c c(b − a) Q. 6. If A, B and C are angles of a triangle, then the
ab − a 2 a − b b 2 − ab = a(b − a) a − b b(b − a) −1 cos C cos B
bc − ac c − a ab − a 2 c(b − a) c − a a(b − a) determinant cos C −1 cos A is equal to
b b−c c cos B cos A −1
= ( b − a )2 a a − b b
(a) 0 (b) –1
c c−a a
(c) 1 (d) None of these
 [on taking (b – a) common from(bC−1 aand
[On taking C3 each]
) common  [NCERT Exemp. Ex. 4.3, Q. 29, Page 81]
from C1 and C3 each]
b−c b−c c
= ( b − a )2 a − b a − b b
DETERMINANTS | 109
Ans. Correct option : (a) 1 1 1
−1 cos C cos B ∆= 1 1+sinq 1
Explanation : We have, cos C −1 cos A 1+cosq 1 1
cos B cos A −1 [Applying C1→C2–C3 and C2→C1–C3]
Applying C1→aC1+bC2+cC3,
0 0 1
− a + b cos C + c cos B cos C cos B = 0 sin q 1
a cos C − b + c cos A −1 cos A cosq 0 1
a cos B + b cos A − c cos A −1
= − sin q .cosq
Also, by projection rule in a triangle, we know that 1
a = b cos C + c cos B, b = c cos A + a cos C and c = − .2 sin q .cosq
2
= a cos B + b cos A 1
= − sin 2q
Using above equation in column first, we get 2
− a + a cos C cos B 0 cos C cos B 1
So, maximum value of ∆ is when sin 2q = −1
b−b −1 cos A = 0 −1 cos A = 0 2
c − c cos A −1 0 cos A −1 0 x−a x−b
[Since, determinant having all elements of any Q. 9. If f ( x ) = x + a 0 x − c , then
column or row gives value of determinant as zero] x+b x+c 0
cos t t 1 (a) f(a) = 0 (b) f(b) = 0
f (t ) (c) f(0) = 0 (d) f(1) = 0
Q. 7. If f(t)= 2 sin t t 2t , then lim 2 is equal to
t →0 t
sin t t t  [NCERT Exemp. Ex. 4.3, Q. 32, Page 82]
Ans. Correct option : (c)
(a) 0 (b) –1
Explanation : Given that,
(c) 2 (d) 3
 [NCERT Exemp. Ex. 4.3, Q. 30, Page 81] 0 x−a x−b
Ans. Correct option : (a) f (x) = x + a 0 x−c
Explanation : We have, x+b x+c 0
cos t t 1
0 x−a x−b
f (t ) = 2 sin t t 2t ,
f (x) = x + a 0 x−c
sin t t t
x+b x+c 0
Expanding along C1,
0 0 a−b 0 0 a−b
= cos t ( t 2 − 2t 2 ) − 2 sin t ( t 2 − t ) + sin t ( 2t 2 − t )
⇒ f ( a) = a + a 0 a − c = 2a 0 a−c
= −t 2 cos t − ( t 2 − t ) 2 sin t + ( 2t 2 − t ) sin t
a+b a+c 0 a+b a+c 0
= – t 2 cos t − t 2 .2 sin t + t.2 sin t + 2t 2 sin t – t sin t
= [( a − b ){2 a.( a + c )}] ≠ 0
= −t 2 cos t + t sin t
0 b−a b−b 0 b−a 0
f (t ) ( −t 2 cos t ) t sin t
∴ lim = lim + lim 2 ∴ f (b ) = b + a 0 b−c = b+a 0 b−c
t →0 t 2 t → 0 t2 t →0 t
sin t b+b b+c 0 2b b + c 0
= − lim cos t + lim
t →0 t →0 t = (b − a)[ 2b(b − c )]
= −1 + 1 = 2b(b − a )(b − c ) ≠ 0
 sin t  0 − a −b
 lim = 1 and cos 0 = 1
 t →0 t  ∴ f ( 0) = a 0 −c
=0 b c 0
1 1 1
      = a (bc) – b(ac)
Q. 8. The maximum value of ∆ = 1 1 + sin θ 1
      = abc – abc
1 + cos θ 1 1       = 0
is (θ is real number)
2 λ −3
1 3
(a) (b) Q. 10. If A = 0 2 5 . Then A–1 exist if
2 2
1 1 3
2 3 (a) λ = 2 (b) λ ≠ 2
(c) 2 (d)
4 (c) λ ≠ –2 (d) None of these
 [NCERT Exemp. Ex. 4.3, Q. 31, Page 81]  [NCERT Exemp. Ex. 4.3, Q. 33, Page 82]
Ans. Correct option : (a) Ans. Correct option : (d)
Explanation : Given that, Explanation : Given that,
110 | OSWAAL NCERT SOLUTIONS – Textbook + Exemplar – MATHEMATICS : Class-XII

2 l −3 x 1 1
A= 0 2 5 ⇒ 0 y 1 =0
1 1 3 −z −z 1 + z
Expanding along R1, Expanding along R1
A = 2 ( 6 − 5 ) − l ( −5 ) − 3 ( −2 ) ⇒ x[ y(1 + z) + z] − 0 + 1( yz) = 0
= 2 + 5l + 6 ⇒ x( y + yz + z) + yz = 0
We know that A–1 exists, if A is non-singular matrix, ⇒ xy + xyz + xz + yz = 0
i.e., |A|≠0 xy xyz xz yz
∴ 2 + 5l + 6 ≠ 0 ⇒ + + + =0
xyz xyz xyz xyz
⇒ 5l ≠ −8 [On dividing (xyz) from both sides]
−8 1 1 1
∴ λ≠ ⇒ +1+ + = 0
5 x y z
−8 1 1 1
So, A−1 exists if and only if λ≠
. ⇒ + + = −1
5 x y z
Q. 11. If A and B are invertible matrices, then which of ∴ x −1 + y −1 + z −1 = −1
the following is not correct?
Q. 13. The value of the determinant
(a) adj. A = A . A−1
x x + y x + 2y
( )
(b) det A−1 =  det ( A ) 
−1
x + 2y x x + y is
x+y x + 2y x
( AB ) = B−1 A−1
−1
(c)
(a) 9x ( x + y)
2
(b) 9y 2 ( x + y )
( A + B ) = B −1 + A −1
−1
(d)
 [NCERT Exemp. Ex. 4.3, Q. 34, Page 82] (c) 3y ( x + y )
2
(d) 7x 2 ( x + y )
Ans. Correct option : (d)  [NCERT Exemp. Ex. 4.3, Q. 36, Page 82]
Explanation : Since, A and B are invertible matrices, Ans. Correct option : (b)
so, we can say that Explanation : Given that,
( AB)−1 = B −1 A −1 ...(i) x x + y x + 2y
1 x + 2y x x+y
Also, A −1 = ( adj A )
A x + y x + 2y x
⇒ adj A = A −1 .| A | ...(ii) Applying C1 → C1 + C2 + C3 and C3 → C3 − C2
Also, −1
det( A ) = [det( A )] −1 3( x + y ) x + y y
1 3( x + y ) x y
⇒ det( A )−1 =
[det( A )] 3( x + y ) x + 2 y −2 y
⇒ det( A ).det( A )−1 = 1 ...(iii) 1 (x + y) y
From equation (iii), we conclude that it is true. = 3( x + y ) 1 x y
1 1 ( x + 2 y ) −2 y
Again, ( A + B)−1 = adj ( A + B)
|( A + B)| [Taking 3(x + y ) common from first column]
⇒ −1
( A + B) ≠ B + A −1 −1
...(iv) 0 y 0
Q. 12. If x, y and z are all different from zero = 3( x + y ) 1 x y [Applying R1 → R1 − R2 ]
1 ( x + 2 y ) −2 y
1+ x 1 1
and 1 1+ y 1 = 0, then value of Expanding along R1
1 1 1+ z = 3( x + y )[ − y( −2 y − y )]
= 3y 2 .3( x + y )
x –1 + y –1 + z –1 is
= 9y 2 (x + y)
(a) xyz (b) x–1y–1z–1
Q. 14. There are two values of a which makes
(c) –x – y – z (d) –1
1 −2 5
 [NCERT Exemp. Ex. 4.3, Q. 35, Page 82]
Ans. Correct option : (d) determinant, ∆ = 2 a –1 = 86. Then sum of
Explanation : We have, 0 4 2a
1+ x 1 1 these numbers is
(a) 4 (b) 5
1 1+ y 1 = 0, (c) –4 (d) 9
1 1 1+ z  [NCERT Exemp. Ex. 4.3, Q. 37, Page 83]
Ans. Correct option : (c)
Applying C1→C1–C3 and C2→C2–C3
Explanation : We have,
DETERMINANTS | 111

1 −2 5 Q. 17. Which of the following is correct?


(a) Determinant is a square matrix.
=∆ 2= a –1 86
(b) Determinant is a number associated to a matrix.
0 4 2a (c) Determinant is a number associated to a square
Expanding along first column matrix.
⇒ 1( 2a 2 + 4) − 2( −4a − 20) + 0 =86 (d) None of these. [NCERT Ex. 4.2, Q. 16, Page 121]
⇒ 2a 2 + 4 + 8a + 40 =86 Ans. Correct option : (c)
Explanation : We know that every square matrix,
⇒ 2a 2 + 8a + 44 − 86 =
0 A = [aij] of order n. We can associate a number
⇒ a 2 + 4a − 21 =
0 called the determinant of a square matrix A, where
aij = (i , j ) element of A.
th
⇒ a 2 + 7 a − 3a − 21 = 0
⇒ ( a + 7 )( a − 3) =
0 Thus, the determinant is a number associated to a
square matrix.
∴ a =−7 , 3
Q. 18. If area of triangle is 35 sq. units with vertices (2,
Required sum =−7 + 3 =−4 –6), (5, 4) and (k, 4). Then k is
x 2 6 2 (a) 12 (b) –2
Q. 15. If = then x is equal to (c) –12, –2 (d) 12, –2
18 x 18 6
 [NCERT Ex. 4.3, Q. 5, Page 123]
(a) 6 (b) ±6
Ans. Correct option : (d)
(c) −6 (d) 0
Explanation : The area of the triangle with vertices
 [NCERT Ex. 4.1, Q. 8, Page 109]
(2, −6), (5, 4) and (k, 4) is given by the relation,
Ans. Correct option : (b)
Explanation : Given that, 2 −6 1
1
x 2 6 2 ∆= 5 4 1
= 2
18 x 18 6 k 4 1
1
⇒ x 2 − 36 = 36 − 36 = [ 2( 4 − 4) + 6(5 − k ) + 1( 20 − 4k )]
2
⇒ x 2 – 36 =
0 1
= [30 − 6k + 20 − 4k ]
⇒ x2 = 36 2
⇒ x= ±6 1
= [50 − 10k ]
Q. 16. Choose the correct answer 2
Let A be a square matrix of order 3 × 3, then kA = 25 − 5k
is equal to It is given that the area of the triangle is ±35.
(a) k A (b) k 2 A Therefore, we have
⇒ 25 − 5k = ±35
(c) k 3 A (d) 3k A
⇒ 5(5 − k ) = ±35
 [NCERT Ex. 4.2, Q. 15, Page 121]
Ans. Correct option : (c) ⇒ 5 − k =±7
Explanation : We know that, A be a square matrix When 5 − k =− 7 , k =5 + 7 =12.
of order 3 × 3 When 5 − k =7 , k =5 − 7 =− 2.
 a1 b1 c1  Hence,= k 12, − 2.
Let A=  a2 b2 c 2 
a11 a12 a13
 a3 b3 c3 
Q. 19. If ∆ = a21 a22 a23 and Aij is cofactors of aij, then
 ka1 kb1 kc1  a31 a32 a33
Then, kA=  ka2 kb2 kc 2  value of D is given by
 ka3 kb3 kc3  (a) a11 A31 + a12 A32 + a13 A33
 ka1 kb1 kc1  (b) a11 A11 + a12 A21 + a13 A31
∴ kA =  ka2 kb2 kc 2  (c) a21 A11 + a22 A12 + a23 A13
 ka3 kb3 kc3  (d) a11 A11 + a21 A21 + a31 A31
a1 b1 c1  [NCERT Ex. 4.4, Q. 5, Page 126]
3 Ans. Correct option : (d)
=k a2 b2 c2
Explanation : We know that :
a3 b3 c3 Δ = Sum of the product of the elements of a column
[Taking out common factor k from each row] (or a row) with their corresponding co-factors
= k 3 | A| =∴ ∆ a11 A11 + a21 A21 + a31 A31
∴ | kA |= k 3 | A | Hence, the value of Δ is given by the expression
given in alternative D.
112 | OSWAAL NCERT SOLUTIONS – Textbook + Exemplar – MATHEMATICS : Class-XII

Q. 20. Let A be a non-singular square matrix of order 3 × Q. 22. Choose the correct answer.
3. Then |adj A| is equal to If a, b, c, are in A.P., then the determinant
(a) A (b) A
2
x + 2 x + 3 x + 2a
(c) A
3
(d) 3 A x + 3 x + 4 x + 2b
 [NCERT Ex. 4.5, Q. 17, Page 132] x + 4 x + 5 x + 2c
Ans. Correct option : (b) (a) 0 (b) 1
Explanation : We know that, (c) x (d) 2x
| A | 0 0   [NCERT Misc. Ex. Q. 17, Page 143]
( adj A ) A =| A | I =  0 | A | 0  Ans. Correct option : (a)
 0 0 | A | Explanation : Given that,
| A| 0 0 x + 2 x + 3 x + 2a
⇒ |( adj A ) A |= 0 | A | 0 ∆ = x + 3 x + 4 x + 2b
0 0 | A| x + 4 x + 5 x + 2c
1 0 0 x+2 x+3 x + 2a
⇒| adj A || A | − | A | 0 1 0 − | A |3 ( I )
3
= x + 3 x + 4 x + (a + c)
0 0 1 x+4 x+5 x + 2c
∴ | adj A |=| A |2 [ 2b = a + c as a , b and c are in A.P.]
Q. 21. If A is an invertible matrix of order 2, then Applying R1 → R1 − R2 and R3 → R3 − R2 , we have
( )
det A–1 is equal to −1 −1 a−c
∆ = x + 3 x + 4 x + (a + c)
1
(a) det ( A ) (b) 1 1 c−a
det( A)
(c) 1 (d) 0 Applying R1 → R1 + R3 , we have
 [NCERT Ex. 4.5, Q. 18, Page 132] 0 0 0
Ans. Correct option : (b) ∆ = x + 3 x + 4 x + (a + c)
Explanation : Given that A is an invertible matrix,
1 1 c−a
1
A −1 exists and A −1 = adj. A. Here, all the elements of the first row (R1) are zero.
| A|
Hence, we have Δ = 0.
a b  Q. 23. Choose the correct answer.
As matrix A is order of 2, let A =  
c d  If x, y, z are non-zero real numbers, then the
 d −b   x 0 0
Then, | A |= ad − bc and adj. A =    
 −c a  inverse of matrix A =  0 y 0  is
Now,  0 0 z 
 d −b   x −1
| A | | A | 0 0   x −1 0 0 
1    
A −1 = adj. A =   (a)  0 y −1 0  (b) xyz  0 y −1 0 
| A|  −c a   0
| A | | A |  0 z −1   0
 0 z −1 
 
d −b x 0 0 1 0 0
1   1  
| A| | A| (c) 0 y 0 (d) 0 1 0
∴ | A −1 |= xyz  xyz 
−c a  0 0 z   0 0 1 
| A| | A|  [NCERT Misc. Ex. Q. 18, Page 143]
Ans. Correct option : (a)
1 d −b Explanation : As we know that from the given
=
| A |2 − c a question,
1
= ( ad − bc ) x 0 0
| A |2 A =  0 y 0 
=
1
.| A |
 0 0 z 
| A |2
∴ | A |= x( yz − 0) = xyz ≠ 0
1
= Now, A11 = yz , A12 = 0, A13 = 0
| A|
1 A21 = 0, A22 = xz , A23 = 0
∴ det( A −1 ) =
    det( A ) A31 = 0, A32 = 0, A33 = xy
 yz 0 0 
∴ adj. A =  0 xz 0 
 0 0 xy 
1
∴ A −1 = adj. A
∴ | A |= x( yz − 0) = xyz ≠ 0
Now, A11 = yz , A12 = 0, A13 = 0
A21 = 0, A22 = xz , A23 = 0
DETERMINANTS | 113
A31 = 0, A32 = 0, A33 = xy
 yz 0 0   1 sin θ 1 
∴ adj. A =  0 xz 0  Q. 24. Let A =  − sin θ 1

sin θ  where 0 0 ≤ θ ≤ 2 π ,
 0 0 xy   −1 − sin θ 1 
1 then
∴ A −1 = adj. A
| A| (a) det ( A ) = 0
 yz 0 0  (b) det ( A ) ∈ ( 2 , ∞ )
1 
= 0 xz 0  (c) det ( A ) ∈ ( 2, 4 )
xyz 
 0 0 xy  (d) det ( A ) ∈ [ 2, 4 ]
 yz   [NCERT Misc. Ex. Q. 19, Page 143]
 0 0  Ans. Correct option : (d)
 xyz  Explanation : As we know that from the question,
 xz 
= 0 0   1 sin q 1 
 xyz 
 A =  − sin q 1 sin q 
xy 
 0 0   −1 − sin q 1 
 xyz 
∴| A |= 1(1 + sin 2 q ) − sin q ( − sin q + sin q ) + 1(sin 2 q + 1)
1 
 0 0 = 1 + sin 2 q + sin 2 q + 1
x 
 1  = 2 + 2 sin 2 q
= 0 0
y = 2(1 + sin 2 q )
 
 1 Now,
 0 0 z  0 ≤ q ≤ 2p
 x −1 0 0 ⇒ −1 ≤ sin q ≤ 1

 
= 0 y −1 0
0 0 z −1 

Very Short Answer Type Questions (1 or 2 mark each)

Q. 1. If A is a matrix of order 3 × 3, then |3A| = _______. Q. 4. If cos 2θ = 0, then


 [NCERT Exemp. Ex. 4.3, Q. 38, Page 83] 0 cos θ sin θ
2

Ans. 3 A = 3 × 3 × 3 A = 27 A . [1] cos θ sin θ 0 = ________


Q. 2. If A is invertible matrix of order 3 × 3, then |A–1|
is equal to _______. sin θ 0 cos θ
 [NCERT Exemp. Ex. 4.3, Q. 39, Page 83]  [NCERT Exemp. Ex. 4.3, Q. 41, Page 83]
−1 1 Ans. Given that, cos 2θ = 0
Ans. | A |= [Since, |A |.| A −1|=1] [1]
| A| p
⇒ cos 2q = cos
Q. 3. If x, y, z ∈ R, then the value of determinant 2
(2 x + 2 − x )2 (2 x − 2 − x )2 1 p
⇒ 2q =
(3 x + 3 − x )2 (3 x − 3 − x )2 1 is equal to _______. 2
p
(4 x + 4 − x )2 (4 x − 4 − x )2 1 ⇒ q=
 [NCERT Exemp. Ex. 4.3, Q. 40, Page 83] 4
Ans. We have, p 1 p 1
∴ sin = and cos =
4 2 4 2
( 2x + 2− x )2 ( 2x − 2− x )2 1 ( 2.2x )( 2.2− x ) ( 2x − 2− x )2 1
2
(3x + 3− x )2 (3x − 3− x )2 1 = ( 2.3x )( 2.3− x ) (3x − 3− x )2 1 1 1
0
( 4x + 4− x )2 ( 4x − 4− x )2 1 ( 2.4x )( 2.4− x ) ( 4x − 4− x )2 1 2 2
[ ( a + b ) − ( a − b ) = 4ab]
2 2 1 1
∴ 0
[ C1 → C1 − C2 ] 2 2
1 1
4 ( 2 x − 2 − x )2 1 0
= 4 (3x − 3− x )2 1
2 2
Expanding along R1 ,
4 ( 4 x − 4 − x )2 1
2
=0  1  1 1  1 
= −   + − 
[Since, C1 and C3 are  2 2 2  2 
2
proportional to each other.]  −2 
[2] = 
2 2 
2
 1 
= − 
 2
1
Expanding along R1 ,
2
 1  1 1  1 
= −  + − 
 2  2 2  2 
114 |
2
OSWAAL −2  SOLUTIONS – Textbook + Exemplar – MATHEMATICS : Class-XII
=  NCERT

2 2 
2
 1  z−x x−z
xyz
= − 
 2 = z−x y−z0
1 z−x z−y 0
=
2
[2] [Taking ( z − x ) common from column first]
Q. 5. If A is a matrix of order 3 × 3, then (A2)–1 1 xyz x − z
= ________.
= ( z − x) 1 0 y−z
 [NCERT Exemp. Ex. 4.3, Q. 42, Page 83]
Ans. A is a matrix of order 3 × 3, then (A2)–1 =(A–1)2 . [1] 1 z−y 0
Q. 6. If A is a matrix of order 3 × 3, then number of Expanding along R1 ,
minors in determinant of A are ________. = ( z − x )[1.{−( y − z)( z − y )} − xyz( z − y ) + ( x − z)( z − y )]
 [NCERT Exemp. Ex. 4.3, Q. 43, Page 83] = ( z − x )( z − y )( − y + z − xyz + x − z)
Ans. A are 9 [Since, in a 3 × 3 matrix, these are 9
= ( z − x )( z − y )( x − y – xyz)
[2]
elements.] [1]
Q. 7. The sum of the products of elements of any row (1 + x )17 (1 + x )19 (1 + x )23
with the co-factors of corresponding elements is Q. 10. If f ( x ) = (1 + x )23 (1 + x )29 (1 + x )34 = A + Bx
equal to _________.
(1 + x )41 (1 + x )43 (1 + x )47
 [NCERT Exemp. Ex. 4.3, Q. 44, Page 83]
Ans. + Cx2 + ....., then A = ______
a11 a12 a13  [NCERT Exemp. Ex. 4.3, Q. 47, Page 84]
Let ∆ = a21 a22 a23 (1 + x )17 (1 + x )19 (1 + x )23
a31 a32 a33 Ans. Given that, f ( x ) = (1 + x )23 (1 + x )29 (1 + x )34
Expanding along R1 , (1 + x )41 (1 + x )43 (1 + x )47
Now,
∆ = a11 A11 + a12 A12 + a13 A13 [1]
1 (1 + x )2 (1 + x )6
= Sum of products of elements of R1with their
corresponding co-factors. f ( x ) = (1 + x ) (1 + x ) (1 + x ) 1 (1 + x )6 (1 + x )11 = 0
17 23 41

x 3 7 1 (1 + x )2 (1 + x )6
Q. 8. If x= –9 is a root of 2 x 2 = 0 then other two [Since R1 and R3 are identical.]
7 6 x [1] A=0
roots are __________. Q. 11. State True or False for the statement :
( ) ( )
−1 3
 [NCERT Exemp. Ex. 4.3, Q. 45, Page 83] A3 = A−1 where A is a square matrix and |A|

x 3 7 ≠ 0. [NCERT Exemp. Ex. 4.3, Q. 48, Page 84]
Ans. Since, 2 x 2 = 0
( ) ( )
−1 n
Ans. True, since, A n = A −1 where n ∈ N. [2]
7 6 x Q. 12. State True or False for the statement :
Expanding along R1 , 1
( aA)−1 = A−1 , where a is any real number and A is
⇒ x ( x 2 − 12 ) − 3 ( 2 x − 14 ) + 7 (12 − 7 x ) = 0 a
a square matrix.
⇒ x 3 − 12 x − 6x + 42 + 84 − 49 x = 0
 [NCERT Exemp. Ex. 4.3, Q. 49, Page 84]
⇒ x 3 − 67 x + 126 = 0 …( i ) Ans. False, since, we know that, if A is a non-singular
Here, 126 × 1 = 9 × 2 × 7 square matrix, then for any scalar a (non-zero). aA
is invertible such that
For x = 2 ,
1   1
⇒ 23 − 67 × 2 + 126 = 134 − 134 = 0 ( aA )  A −1  =  a ,  ( A. A −1 )
 a   a
Hence, x = 2 is a root.
1  1
For x = 7, i.e., ( aA ) is inverse of  A −1  or ( aA )−1 = A −1 ,
a  a
⇒ 73 − 67 × 7 + 126 = 469 − 469 = 0
where ' a ' is any non-zero scalar.
Hence, x = 7 is also a root. In the above statement, a is any real number. [2]
[2]
Q. 13. State True or False for the statement :
0 xyz x − z –1
A–1 ≠ A , where A is non-singular matrix.

Q. 9. Evaluate y − x 0 y − z = ________
 [NCERT Exemp. Ex. 4.3, Q. 50, Page 84]
z−x z−y 0 –1
Ans. False, A ≠ A , where A is non-singular
–1
 [NCERT Exemp. Ex. 4.3, Q. 46, Page 83]
matrix. [2]
0 xyz x−z
Q. 14. State True or False for the statement :
Ans. Given that, y − x 0 y−z If A and B are matrices of order 3 and |A| = 5, |B|
z−x z−y 0 = 3, then
Applying C1→C1–C3, we get |3AB| = 27 × 5 × 3 = 405.
 [NCERT Exemp. Ex. 4.3, Q. 51, Page 84]
DETERMINANTS | 115
Ans. True Ans. True,
We know that, sin A cos A sin A + cos B
AB = A . B Since, sin B cos A sin B + cos B
3 AB = 27 AB sin C cos A sin C + cos B
= 27 A . B sin A cos A sin A sin A cos A cos B
= 27 × 5 × 3 = sin B cos A sin B + sin B cos A cos B
= 405 [2] sin C cos A sin C sin C cos A cos B
Q. 15. State True or False for the statement :
sin A cos A cos B
If the value of a third-order determinant is 12, then
the value of the determinant formed by replacing = 0 + sin B cos A cos B
each element by its co-factor will be 144. sin C cos A cos B
 [NCERT Exemp. Ex. 4.3, Q. 52, Page 84] [Since, in the first determinant C1 and C3
Ans. True, let A is the determinant. are identicals.]
∴|A|= 12
sin A 1 1
Also, we know that, if A is a square matrix of order
n, then adj A =| A |n −1 = cos A.cos B sin B 1 1
sin C 1 1
For n = 3,| adj A |=| A |3−1 =| A |2
[Taking cos A common from C2 and cos B common
= (12)2 = 144 [2] from C3]
Q. 16. State True or False for the statement :
=0 [Since, C2 and C3 are identical] [2]
x+1 x+2 x+a Q. 19. State True or False for the statement :
x+2 x+3 x+b =0 x+a p+u l+ f
x+3 x+4 x+c If the determinant y + b q + v m + g splits
where a, b and c are in A.P. z+ c r + w n+ h
 [NCERT Exemp. Ex. 4.3, Q. 53, Page 84]
into exactly K determinants of order 3, each
Ans. True,
element of which contains only one term, then the
Since, a, b and c are in AP, then 2b = a + c
value of K is 8.
x +1 x + 2 x + a
 [NCERT Exemp. Ex. 4.3, Q.56, Page 84]
∴ x+2 x+3 x+b =0 Ans. True,
x+3 x+4 x+c x+a p+u l+ f
[  R1→R1+R3] Given that, y + b q + v m + g
2x + 4 2x + 6 2x + a + c z+c r+w n+h
⇒ x+2 x+3 x+b =0
x p l a u f
x+3 x+4 x+c
⇒ y+b q+v m+ g + y+b q+v m+ g
[  2b = a + c ]
z+c r+w n+h z+c r+w n+h
2( x + 2) 2( x + 3) 2( x + b )
[Spliting first row]
⇒ x+2 x+3 x+b =0
x p l x p l
x+3 x+4 x+c
⇒ y q m + b v g
⇒ 0=0
z+c r+w n+h z+c r+w n+h
[Since Rl and R2 are proportional to each other.]
[2] a u f a u f
Q. 17. State True or False for the statement : + y q m + b v g
| adj. A|=| A|2 where A is a square matrix of order z+c r+w n+h z+c r+w n+h
two. [NCERT Exemp. Ex. 4.3, Q. 54, Page 84] [Spliting second row]
Ans. False,
If A is a square matrix of order n, then Similarly, we can split these 4 determinants
in 8 determinants by splitting each one in two
| adj. A |=| A |n −1
determinants further. [2]
⇒| adj. A |=| A |2 −1 =| A | [ n = 2] [2] Q. 20. State True or False for the statement :
Q. 18. State True or False for the statement :
a p x p+ x a+ x a+ p
sin A cos A sin A + cos B
Let ∆ = b q y = 16, then ∆ 1 = q + y b + y b + q = 32
The determinant sin B cos A sin B + cos B is\ c r z r+ z c+ z c+r

sin C cos A sin C + cos B
equal to zero.  [NCERT Exemp. Ex. 4.3, Q. 57, Page 85]
 [NCERT Exemp. Ex. 4.3, Q. 55, Page 84] Ans. True,
116 | OSWAAL NCERT SOLUTIONS – Textbook + Exemplar – MATHEMATICS : Class-XII

a p x 2 4
Q. 22. Evaluate the determinants
We have ∆ = b q y = 16 −5 −1
c r z  [NCERT Ex. 4.1, Q. 1, Page 108]
Ans. 2 4 [1]
p+x a+x a+p = 2( −1) − 4( −5)
−5 −1
and we have to prove, ∆1 = q + y b + y b + q = 32
= − 2+20
r+z c+z c+r
=18
Applying Cl → C1 + C2 + C3 Q. 23. Evaluate the determinants
2 p + 2x + 2a a + x a + p cos θ − sin θ
(i)
∆1 = 2q + 2 y + 2b b + y b + q sin θ cos θ
2r + 2 z + 2 c c + z c + r
(ii) x − x + 1 x − 1  [NCERT Ex. 4.1, Q. 2, Page 108]
2

[Taking 2 common from C1 and then x+1 x+1


C1 → C1 − C2 , C2 → C2 − C3 ]
Ans. (i) cosq − sin q
p x−p a+p = (cosq )(cosq ) − ( − sin q )(sin q )
sin q cosq
=2 q y−q b+q
= cos2 q + sin 2 q
r z−r c+r
=1
p x a+p p p a+p x2 − x + 1 x − 1
(ii)
=2 q y b+q − q q b+q x +1 x +1
r z c+r r r c+r
= ( x 2 − x + 1)( x + 1) − ( x − 1)( x + 1)
[Since, two columns C1 and C2 are identicals]
= x 3 − x 2 + x + x 2 − x + 1 − ( x 2 − 1)
p x a+p
= x3 + 1 − x 2 + 1
=2 q y b+q −0
[2] = x3 − x 2 + 2
r z c+r
 1 2
[Since, Cl and C3 are identical in second determinant Q. 24. If A =   then show that 2 A = 4 A .
and in first determinant, C1 ↔ C2 and then C1 ↔ C3 ] 4 2
 [NCERT Ex. 4.1, Q. 3, Page 108]
p x a p x p Ans. The given matrix is
=2 q y b +2 q y q
1 2
r z c r z r A=
      4 2 
a p x
1 2 2 4
=2 b q y +0 ∴ 2A = 2  =
4 2   8 4 
c r z
2 4
  = 2 16   [  ∆= 16] LHS = 2 A =
   = 32 8 4
Hence proved. [2] = 2× 4 − 4×8
Q. 21. State True or False for the statement :
= 8 − 32
1 1 1
= −24
The maximum value of 1 1 + sin θ 1
1 2
1 1 1 + cos θ Now, A =
4 2
1
is .  [NCERT Exemp. Ex. 4.3, Q. 58, Page 85] = 1× 2 − 2 × 4
2
Ans. True, = 2−8
Since, = −6
1 1 1 ∴ RHS = 4 A = 4 × ( −6) = −24
0 sin q 0 [ R2 → R2 − R1 and R3 → R3 − R1 ] ∴ LHS = RHS [2]
Q. 25. Evaluate the determinants
0 0 cosq
On expanding along third row, we get the value of 3 −1 −2 3 −4 5
the determinant (i) 0 0 −1 (ii) 1 1 −2
= cosq .sin q 3 −5 0 2 3 1
1
= sin 2q 0 1 2 2 −1 −2
2
1 (iii) −1 0 −3 (iv) 0 2 −1
=
2 −2 3 0 3 −5 0
when q is 45° which gives maximum value. [2]  [NCERT Ex. 4.1, Q. 5, Page 108]
1 1−3 −3 2 2−3 −3 2 DETERMINANTS
21 1 | 117
A =A1 = 1 −1 −1 − 2− 2
4 4−9 −9 5 5−9 −9 5 54 4
3 −1 −2 = 1(=−91(+−912+)12 − 1)(− 181( −+18 − 2) (−82−(85)− 5)
15+)15
Ans. (i) Let A = 0 0 −1 = 1(=3)1(−31)(− 31()−−32) (−3)2(3)
3 −5 0 = 3 =+ 3 +− 36 − 6
It can be observed that in the second row, two = 6 =− 6 − 6
entries are zero. Thus, we expand along the second = 0= 0 [2]
row for easier calculation. Q. 27. Find values of x, if
−1 −2 3 −2 3 −1 2 4 2x 4
A = −0 +0 − ( −1) (i) =
−5 0 3 0 3 −5 5 1 6 x
= ( −15 + 3)
(ii) 2 3 = x 3  [NCERT Ex. 4.1, Q. 7, Page 109]
= −12 [2] 4 5 2x 5
3 −4 5
Ans. (i) 2 4 2x 4 [1]
(ii) Let A = 1 1 −2 =
5 1 6 x
2 3 1
⇒ 2 × 1 − 5 × 4 = 2x × x − 6 × 4
By expanding along the first row, we have
⇒ 2 − 20 = 2x 2 − 24
1 −2 1 −2 1 1
A =3 +4 +5 ⇒ 2x 2 = 6
3 1 2 1 2 3
⇒ x2 = 3
= 3(1 + 6) + 4(1 + 4) + 5(3 − 2)
= 3( 7 ) + 4(5) + 5(1) ⇒ x=± 3
= 21 + 20 + 5 (ii) 2 3 x 3 [1]
=
= 46 4 5 2x 5
[2]
⇒ 2 × 5 − 3 × 4 = x × 5 − 3 × 2x
0 1 2
⇒ 10 − 12 = 5x − 6x
(iii) Let A = −1 0 −3
⇒ −2 = − x
−2 3 0
⇒ x=2
By expanding along the first row, we have Q. 28. Using the property of determinants and without
0 −3 −1 −3 −1 0 x a x+a
A =0 −1 +2
3 0 −2 0 −2 3 expanding, prove that y b y + b = 0
= 0 − 1( 0 − 6) + 2( −3 − 0) z c z+c
= −1( −6) + 2( −3)  [NCERT Ex. 4.2, Q. 1, Page 119]
=6−6 x a x+a x a a x a x
=0 [2] Ans. y b y+b = y b b + y b y =0+0=0
2 −1 −2 z c z+c z c c z c z
(iv) Let A = 0 2 −1 Here, the two columns of the determinant are
3 −5 0 identical. [2]
Q. 29. Using the property of determinants and without
By expanding along the first column, we have
a−b b− c c− a
2 −1 −1 −2 −1 −2 expanding, prove that b − c c − a a − b = 0
A =2 −0 +3
−5 0 −5 0 2 −1 c− a a−b b− c
= 2( 0 − 5) − 0 + 3(1 + 4)  [NCERT Ex. 4.2, Q. 2, Page 119]
= −10 + 15
a−b b−c c−a
=5 [2] Ans. Let we assume that ∆= b − c c − a a − b = 0
 1 1 −2  c−a a−b b−c
 
Q. 26. If A =  2 1 −3  Find |A|. Applying R1→R1+R2, we have
 5 4 −9 
a−c b−a c−b
 [NCERT Ex. 4.1, Q. 6, Page 109]
∆= b−c c−a a−b
1 1 −2 
Ans. Let A =  2 1 −3 −( a − c ) −(b − a) −( c − b )
 5 4 −9  a−c b−a c−b
By expanding along the first row, we have = b−c c−a a−b
1 −3 2 −3 2 1 ( a − c ) (b − a) ( c − b )
A =1 −1 −2 Here, the two rows R1 and R3 are identical.
4 −9 5 −9 5 4
Therefore, △=0 [2]
= 1( −9 + 12) − 1( −18 + 15) − 2(8 − 5)
= 1(3) − 1( −3) − 2(3)
= 3+3−6
1 0 1
1
∆= 6 0 1
2
4 3 1
118 | OSWAAL NCERT SOLUTIONS – Textbook + Exemplar – MATHEMATICS1 : Class-XII
= [1( 0 − 3) − 0( 6 − 4) + 1(18 − 0)]
2
Q. 30. Using the property of determinants and without 1
= [ −3 + 18]
2 7 65 2
expanding, prove that 3 8 75 = 0 15
= square units
5 9 86 2 [2]
 [NCERT Ex. 4.2, Q. 3, Page 119] (ii) The area of the triangle with vertices (2, 7), (1, 1)
Ans. and (10, 8) is given by the relation,
2 7 65 2 7 63+2 2 7 1
1
3 8 75 = 3 8 72+3 ∆= 1 1 1
2
5 9 86 5 9 81+5 10 8 1
1
2 7 63 2 7 2 = [ 2(1 − 8) − 7(1 − 10) + 1(8 − 10)]
2
= 3 8 72 + 3 8 3
1
5 9 81 5 9 5 = [ 2( −7) − 7( −9) + 1( −2)]
2
2 7 9(7) 1
= [ −14 + 63 − 2]
= 3 8 9(8) + 0 2
5 9 9(9) 1
= [ −16 + 63]
[Two coulmns are identical.] 2
47
2 7 7 = square units
2 [2]
=9 3 8 8
(iii) The area of the triangle with vertices (−2, −3), (3, 2)
5 9 9 and (−1, −8) is given by the relation,
=0 −2 −3 1
1
[Two coulmns are identical.]  [2] ∆= 3 2 1
2
Q. 31. Using the property of determinants and without −1 −8 1
1 bc a( b + c ) 1
expanding, prove that 1 ca b( c + a) = 0 = [ −2( 2 + 8) + 3(3 + 1) + 1( −24 + 2)]
2
1 ab c( a + b) 1
= [ −2(10) + 3( 4) + 1( −22)]
 [NCERT Ex. 4.2, Q. 4, Page 119] 2
1 bc a( b + c ) 1
= [ −20 + 12 − 22]
Ans. Let we assume that, ∆ = 1 ca b( c + a ) = 0 2
30
1 ab c( a + b ) =−
2
By applying C3→C3+C2, we get
= −15 square units
1 bc ab + bc + ca
Hence, the area of the triangle is |–15|=15 square
⇒ ∆ = 1 ca ab + bc + ca = 0 units. [2]
1 ab ab + bc + ca 1 2
Q. 33. Find adjoint of each of the matrices  .
1 bc 1 3 4
⇒ ( ab + bc + ca ) 1 ca 1 = 0  [NCERT Ex. 4.5, Q. 1, Page 131]
1 ab 1 1 2 
Ans. Let A =  .
Here, two columns C1 and C3 are proportional. 3 4 
⇒ Δ=0 [2] We have,
Q. 32. Find area of the triangle with vertices at the point A11 = 4, A12 = −3, A21 = −2, A22 = 1
given in each of the following A12  '  4 −2
A
(i) (1, 0), (6, 0), (4, 3) ∴ adj. A =  11  =  −3 1  .
A
 21 A 22    [2]
(ii) (2, 7), (1, 1), (10, 8)
(iii) (–2, –3), (3, 2), (–1, –8) Q. 34. Find the inverse of each of the matrices (if it exists)
 [NCERT Ex. 4.3, Q. 1, Page 122]  2 −2 
4 3  .
Ans. (i) The area of the triangle with vertices (1, 0), (6, 0)   [NCERT Ex. 4.5, Q. 5, Page 132]
and (4, 3) is given by the relation,
Ans.  2 −2
1 0 1 Let A =  .
1 4 3 
∆= 6 0 1
2 We have
4 3 1
| A | = 6 + 8 = 14
1 Now ,
= [1( 0 − 3) − 0( 6 − 4) + 1(18 − 0)]
2
A11 = 3, A12 = −4, A21 = 2, A22 = 2
1
= [ −3 + 18]
2
15
= square units
2
DETERMINANTS | 119

 A11
'
A12   3 2  Thus, A–1 exists.
∴ adj. A =  =  Therefore, the given system of two equations will
 A21 A22  −4 2 
be consistent. [2]
1 1  3 2 Q. 38. Examine the consistency of the system of
∴ A−1 = adj. A =  
| A| 14−4 2  equations.
 [2]
x + 3y =5
Q. 35. Find the inverse of each of the matrices (if it exists)
2x + 6y = 8 [NCERT Ex. 4.6, Q. 3, Page 136]
 −1 5 
 −3 2  .  [NCERT Ex. 4.5, Q. 6, Page 132] Ans. The given system of equations is
 
x + 3y =5
 −1 5 
Ans. Let A =  . 2x + 6 y =
8
 −3 2  The given system of equations can be written in the
We have, form of AX = B, where
| A |=−2 + 15 =13 1 3 x  5 
= A =  , X =  and B  
Now, 2 6 y 8 
A11 =
2, A12 =
3, A21 =
−5, A22 =
−1 Now,
'
A A12   2 −5 |=
A | 1( 6) − 3( 2)
∴ adj
= . A  11 =  
 A21 A22   3 −1 = 6−6
1 1 2 −5 =0
∴ A= −1
adj=
.A  ∴ A is a singular matrix.
| A| 13  3 −1 [2] Now,
Q. 36. Examine the consistency of the system of  6 −3
equations. ( adj. A ) =  
x + 2y = 2  −2 1 
 6 −3 5 30 − 24   6 
2 x + 3y = 3 [NCERT Ex. 4.6, Q. 1, Page 136] =
( adj. A )B  =
   =   ≠0
 −2 1  8  −10 + 8  −2 
Ans. The given system of equations is : Thus, the solution of the given system of equations
x + 2y = 2 does not exist. Hence, the system of equations is
inconsistent. [2]
2x + 3y =3 Q. 39. Solve system of linear equations, using matrix
The given system of equations can be written in the method.
form of AX = B, where 5x + 2y = 3
1 2  x  2 3x + 2y = 5 [NCERT Ex. 4.6, Q. 10, Page 136]
A=  , X=  y  and B=  3  Ans. The given system of equations can be written in the
2 3    
form of AX = B, where
Now,
5 2  x   3
| A |=1(3) − 2(2) A=  , X=  y  and B= 5
 3 2     
=3 − 4
=− 1 ≠ 0 Now,
Hence, A is non-singular. |A|= 10 − 6 = 4 ≠ 0
Thus, A–1 exists. Thus, A is non-singular. Therefore, its inverse
Therefore, the given system of two equations will exists. [2]
be consistent. [2] Now A11 = 2, A12 = –3, A21= –2, A22 = 5
'
\ Adj. A =  A12  =  2
Q. 37. Examine the consistency of the system of A -2 
equations.
A11 A   -3 5 
 21 22 
2x − y = 5 1
x+y= 4  [NCERT Ex. 4.6, Q. 2, Page 136] ⇒ A–1 = ( adj A ) = 1  -23 -2 
5 
A 4
Ans. The given system of equations is : 1 1
2x − y =5  − 
2 2
x+y= 4 =
 −3 5 
The given system of equations can be written in the 
 4 4 


form of AX = B, where X =A B–1

 2 −1 x  5 1 −1 
= A = , X =  and B  4   
1 1  y
    2 2 3
= −3 5  
Now,   5 
| A=| 2(1) − ( −1)(1) 
 4 4 

= 2 +1  -1
X= 4
= 3≠0  
Hence, A is non-singular. x = 1, y = 4
120 | OSWAAL NCERT SOLUTIONS – Textbook + Exemplar – MATHEMATICS : Class-XII

1 x y 0 xy 2 xz 2
Q. 40. Evaluate 1 x + y y Ans. We have, x 2 y 0 yz 2
1 x x+y x 2 z zy 2 0
 [NCERT Misc. Ex. Q. 10, Page 142] Taking x 22 , y 22 and z 22 common from C11 , C22
Ans. 1 x y and C33 , respectively.

∆= 1 x+y y 0 x x
1 x x+y =x y z y 0 y
22 22 22

Applying R2 → R2 − R1 and R3 → R3 − R1 , we have z z 0


1 x y Applying C22 → C22 − C33
∆= 0 y 0 0 0 x
0 0 x = x y z y −y y
2 22 22
2

Expanding along C1 , we have z z 0


∆ = 1( xy − 0) = xy [2] = x 22 y 22 z 22 [ x( yz + yz)]
Q. 41. Using the properties of determinants, evaluate = x 22 y 22 z 22 .2 xyz
x2 − x + 1 x − 1 = 2 x 33 y 33 z 33 [2]
x+1 x+1
Q. 44. Using the properties of determinants, evaluate
 [NCERT Exemp. Ex. 4.3, Q. 1, Page 77] x+4 x x
Ans. We have, x − x + 1 x − 1
2
x x+4 x
x +1 x +1 x x x+4
.
Applying C1 → C1 − C2  [NCERT Exemp. Ex. 4.3, Q. 5, Page 77]
x 2 − 2x + 2 x − 1 x+4 x x
0 x +1 Ans. We have, x x+4 x
On expanding, we get x x x+4
=(x 2 − 2x + 2).( x + 1) − ( x − 1).0 Applying R1 → R1 + R2 + R3
=x 3 − 2x 2 + 2x + x 2 − 2x + 2 3x + 4 3x + 4 3x + 4
= x3 − x 2 + 2 [2] = x x+4 x
Q. 42. Using the properties of determinants, evaluate x x x+4
a+ x y z
1 1 1
x a+ y z .
= (3x + 4) x x + 4 x
x y a+ z
x x x+4
 [NCERT Exemp. Ex. 4.3, Q. 2, Page 77]
Applying C1 → C1 − C2 , C2 → C2 − C3
a+x y z
0 0 1
Ans. We have x a+y z
= (3x + 4) −4 4 x
x y a+z
0 −4 x + 4
On applying R1 → R1 – R2 and R2 → R2 – R3
= 16(3x + 4) [2]
a −a 0
Q. 45. Using the properties of determinants, prove that
= 0 a −a
a2 + 2 a 2 a + 1 1
x y a+z
2 a + 1 a + 2 1 = ( a − 1)3
Applying C2 → C2 + C1
3 3 1
a 0 0 .
= 0 a −a  [NCERT Exemp. Ex. 4.3, Q. 9, Page 78]
Ans. Given that,
x x+y a+z
a 2 + 2a 2a + 1 1
= a( a 2 + az + ax + ay )
2a + 1 a+2 1 = ( a − 1)3
= a2 (a + z + x + y)
 [2] 3 3 1
Q. 43. Using the properties of determinants, evaluate
Applying R1 → R1 − R2 and R2 → R2 − R3
0 xy 2 xz 2
2
x y 0 yz 2 a2 − 1 a − 1 0
x 2 z zy 2 0 . = 2a − 2 a − 1 0
 [NCERT Exemp. Ex. 4.3, Q. 3, Page 77] 3 3 1

Taking (a − 1) common from R1 and R2
a +1 1 0
= ( a − 1) 2 2
1 0
Applying R1 → R1 − R2 and R2 → R2 − R3
a2 − 1 a − 1 0
= 2a − 2 a − 1 0  2 −3 DETERMINANTS | 121
Given, A =  
3 3 1  −4 7 
Taking (a − 1) common from R1 and R2
−1 1 7 3  1 7 3 
a +1 1 0 ⇒ A −1 =  =  
14 − 12  4 2  2  4 2 
= ( a − 1)2 2 1 0 Now,
3 3 1
LHS=2A −−11
Expanding alone R3
7 3 
= ( a − 1)2 [1.( a + 1) − 2] =2. 1  
2 4 2 [2]
= ( a − 1)3 [2]
Q. 46. If a1, a2, a3, ..., av ...are in G.P., then prove that the 7 3 
= 
ar + 1 ar + 5 ar + 9 4 2
a ar + 11 ar + 15 is independent RHS=9I − A
determinant r +7

of r. ar + 11 ar + 17 ar + 21 1 0   2 −3 
=9  − 
 [NCERT Exemp. Ex. 4.3, Q. 14, Page 78] 0 1   −4 7 
a = AR( r +1) −1 = AR r  9−2 0 − ( −3)
Ans. We know that, r +1 = 
where r = rth term of a GP, A = First term of a GP 0 − ( −4) 9 − 7 
and R = Common difference of GP 7 3 
= 
a r +1 a r + 5 a r + 9
4 2
We have, ar + 7 ar +11 ar +15 Since, LHS=RHS
ar +11 ar +17 ar + 21 Hence proved. [2]
AR r AR r + 4 AR r + 8  3 −1
= AR r + 6 AR r +10 AR r +14 Q. 49. If |A| = 3 and A =  5 2  then write the adj. A.
−1
− 
AR r +10 AR r +16 AR r + 20  3 3 
1 R4 R8  [CBSE Board, Foreign Scheme, 2017]
 3 −1
= AR r . AR r + 6 . AR r +10 1 R 4 R8  9 −3
Ans. adj. A = 3  5 2  =  [1]
1 R6 R10 −   −5 2 
 3 3 
[Taking AR r , AR r + 6 and AR r +10 common from the
row R1 , R2 and R3 , respectively] 1 1 1
=0 [Since, R1 and R2 are identicals] Q. 50. Find the maximum value of 1 1 + sin θ 1 .
[2]
Q. 47. Show that the points (a + 5, a – 4), (a – 2, a + 3) and 1 1 1 + cos θ
(a, a) do not lie on a straight line for any value of a.  [CBSE Board, Delhi Region, 2016]
 [NCERT Exemp. Ex. 4.3, Q. 15, Page 78] Ans. Let we assume that,
Ans. Given, the points are
1 1 1
( a + 5, a – 4 ) , ( a – 2, a + 3) and ( a, a )
∆ = 1 1 + sin q 1 = sin q .cosq [½]
a+5 a−4 1
1 1 1 1 + cosq
∆= a−2 a+3 1
2 1  1
a a 1 = sin 2q ∴ Maximum value = 2  [½]
2  
Applying Rl → R1 − R3 and R2 → R2 − R3
x sin θ cos θ
5 −4 0
1 Q. 51. Write the value of x. If − sin θ − x 1 = 8,
= −2 3 0
2 cos θ 1 x
a a 1 write the value of x.
1  [CBSE Board, Foreign Scheme, 2016]
= [1(15 − 8)]
2 Ans. On expanding we get
7 ⇒ x[ − x 2 − 1] − sin θ [ − x sin θ − cosθ ] + cos θ [ − sin θ
= ≠0
2 + x.cos θ ] = 8
Hence, given points form a triangle, i.e., points do
not lie in a straight line. [2] ⇒ − x 3 − x + x.sin 2 q + sin q .cosq − sin q .cosq
 2 −3  −1 + x.cos2 q = 8
Q. 48. Given A =   Compute A and show that
 − 4 7  But we know that cos2q + sin 2 q = 1
−1
2 A = 9 I − A.  [CBSE Board, Delhi Region, 2018] So,
Ans.
⇒ −x 3 − x + x = 8
 2 −3
Given, A =   ⇒ x 3 = −8
 −4 7 
⇒ x = −2 [1]
1 7 3  1 7 3 
⇒ A −1 =  =  
14 − 12  4 2  2  4 2 
Now,
−1
122 | OSWAAL NCERT SOLUTIONS – Textbook + Exemplar – MATHEMATICS : Class-XII

Short Answer Type Questions (3 or 4 marks each)

1 0 1 Q. 3. Using the property of determinants and without


  − a2 ab ac
Q. 1. If A =  0 1 2  then show that 3 A = 27| A |.
 0 0 4  expanding, prove that ba − b2 bc = 4 a2 b2 c 2 .
 [NCERT Ex. 4.1, Q. 4, Page 108] ca cb − c 2
1 0 1   [NCERT Ex. 4.2, Q. 7, Page 120]
  Ans. Taking LHS and we get
Ans. The given matrix is A = 0 1 2 
0 0 4  −a 2 ab ac
It can be observed that in the first column, two ∆ = ba −b 2 bc
entries are zero. Thus, we expand along the first
ca cb −c 2
column (C1) for easier calculation.
1 2 0 1 0 1 −a b c
| A|= 1 −0 +0 = abc a −b c
0 4 0 4 1 2
=1(4 − 0) − 0+0 a b −c
=4 [Taking out factors a , b , c from R1 , R2 and R3 ]
Therefore, 27|A|=27(4)=108 ...(i) −1 1 1
1 0 1   3 0 3 =a 2b 2 c 2 1 −1 1
Now, 3A=3 0 1 2  = 0 3 6  1 1 −1
0 0 4  0 0 12  [Taking out factors a , b , c from C1 , C 2 and C3 ]
3 6 0 3 0 3 Applying R2→R2+R1 and R3→R3+R1 we have
Therefore, |3A|=3 +0 +0
0 12 0 12 3 6 −1 1 1
=3(36 − 0)=3(36)=108 ...(ii) ∆= 0 0 2
From equations (i) and (ii), we get 0 2 0
|3A|= 27| A|  0 2
= a 2b 2 c 2 ( −1)
Hence proved. [3] 2 0
Q. 2. Using the property of determinants and without = − a 2b 2 c 2 ( 0 − 4)
0 a −b =4a 2b 2 c 2
expanding, prove that − a 0 − c = 0 Hence proved. [3]
b c 0 Q. 4. (i) Find equation of line joining (1, 2) and (3, 6)
 [NCERT Ex. 4.2, Q. 6, Page 120] using determinants.
Ans. We have, (ii) Find equation of line joining (3, 1) and (9, 3) using
determinants. [NCERT Ex. 4.3, Q. 4, Page 123]
0 a −b
Ans. (i) Let P(x, y) be any point on the line joining
∆ = −a 0 −c = 0 points A(1, 2) and B(3, 6). Then, the points A, B and
b c 0 P are collinear. Therefore, the area of triangle ABP
Applying R1 → cR1 , we get will be zero.
0 ac −bc 1 2 1
1 1
∆= − a 0 − c ∴ 3 6 1 =0
c 2
b c 0 x y 1
1
Applying R1 → R1 − bR2 , we get ⇒ [1(6 − y ) − 2(3 − x ) + 1(3y − 6x )] = 0
2
ab ac 0 ⇒ 6 − y − 6 + 2x + 3y − 6x = 0
1
∆= − a 0 − c ⇒ 2 y − 4x = 0
c
b c 0 ⇒ y = 2x
b c 0 Hence, the equation of the line joining the given
a points is y = 2x. [3]
= −a 0 −c
c (ii) Let P(x, y) be any point on the line joining points
b c 0
A(3, 1) and B(9, 3). Then, the points A, B and P are
Here, the two rows R1 and R3 are identical. collinear. Therefore, the area of triangle ABP will be
So, Δ = 0. [3] zero.
DETERMINANTS | 123

3 1 1 5 3 8
1
∴ 9 3 1 =0 Ans. The given determinant is ∆ = 2 0 1
2
x y 1 1 2 3
1 We have,
⇒ [3(3 − y ) − 1(9 − x ) + 1(9 y − 3x )] = 0 3 8
2
M21= = 9 − 16 = −7
⇒ 9 − 3y − 9 + x + 9 y − 3x = 0 2 3
⇒ 6 y − 2x = 0
Therefore, A21 = co-factor of a21 = ( −1)
2 +1
M21 = 7
⇒ x − 3y = 0
5 8
Hence, the equation of the line joining the given M22= = 15 − 8 = 7
1 3
points is x − 3y = 0. [3]
Therefore, A22 = co-factor of a22 = ( −1)
2 + 2
Q. 5. Write Minors and Co-factors of the elements of M22 = 7
following determinants : 5 3
2 −4 a c M23 = = 10 − 3 = 7
(i) (ii) 1 2
0 3 b d
Therefore, A23 = co-factor of a23 = ( −1)
2 + 3
M23 = −7
 [NCERT Ex. 4.4, Q. 1, Page 126]
We know that Δ is equal to the sum of the product
Ans. (i) The given determinant is 2 −4 of the elements of the second row with their
0 3 corresponding co-factors.
Minor of element aij is Mij . Therefore, ∆ = a21 A21 + a22 A22 + a23 A23
Therefore, = 2 ( 7 ) + 0 ( 7 ) + 1( −7 )
M11 = Minor of element a11 = 3 = 14 – 7
M12 = Minor of element a12 = 0 = 7 [3]
M21 = Minor of element a21 = − 4  1 −1 2 
M22 = Minor of element a22 = 2 Q. 7. Find adjoint of each of the matrices  2 3 5 
Co-factor of aij is Aij = ( −1) i + j Mij .  −2 0 1 

Therefore,  [NCERT Ex. 4.5, Q. 2, Page 131]

A11 = ( −1)
1 +1
M11 = ( −1) ( 3) = 3
2  1 −1 2 
Ans. Let we assume that, A =  2 3 5 
A12 = ( −1) M12 = ( −1) ( 0 ) = 0
1+ 2 3

 −2 0 1 
A21 = ( –1) M21 = ( −1)3 ( −4 ) = 4
2 +1
We have,
A22 = ( −1) M22 = ( −1) ( 2)
2 + 2 4
2 3 5
[3] A11 = = 3−0 = 3
0 1
a c
(ii) The given determinant is 2 5
b d A12 = − = −( 2 + 10) = −12
Minor of element aij is Mij . −2 1

Therefore, 2 3
A13 = =0+6=6
M11 = Minor of element a11 =d . −2 0

M12 = Minor of element a12 = b −1 2


A21 = − = −( −1 − 0) = 1
M21 = Minor of element a21 = c 0 1
M22 = Minor of element a22 = a 1 2
A 22 = =1+ 4 = 5
−2 1
Co-factor of aij is Aij = ( −1)
i +j
Mij .
1 −1
Therefore, A23 = − = −( 0 − 2) = 2
−2 0
A11 = ( −1) M11 = ( −1) ( d ) = d
1+1 2
−1 2
A 31 = = −5 − 6 = −11
A12 = ( −1) M12 = ( −1) ( b ) = − b
1+ 2 3
3 5
A21 = ( −1) M21 = (1) ( c ) = − c
2 +1 3
1 2
A32 = − = −(5 − 4) = −1
2 5
A22 = ( −1) 2 + 2 M22 = ( −1) ( a ) = a
4
[3]
1 −1
Q. 6. Using Co-factors of elements of second row, A 33 = = 3+ 2 = 5
5 3 8 2 3
evaluate ∆ = 2 0 1  A11 A21 A31 
1 2 3 Hence, adj. A=  A12 A22 A32 
 [NCERT Ex. 4.4, Q. 3, Page 126]  A13 A23 A33 
 3 1 −11
=  −12 5 −1 
 6 2 5 
 A11 A21 A31 
 
124 | Hence, adj .
OSWAAL NCERT A=  A12SOLUTIONS
A22 A32  – Textbook + Exemplar – MATHEMATICS : Class-XII
 A13 A23 A33 
 3 1 −11 Q. 10. Find the inverse of each of the matrices (if it exists)
 12 5 1  1 0 0 
=  − − 
 6 2 5   
3 3 0 .  [NCERT Ex. 4.5, Q. 8, Page 132]
[3]
Q. 8. Verify A(adj. A) = (adj A) A = |A|I.  5 2 −1
2 3 Ans. Let we assume that,
 -4 –6  1 0 0 
[NCERT Ex. 4.5, Q. 3, Page 131]
Ans. Let we assume that, A = 3 3 0 
2 3 5 2 −1
A=  We have,
 −4 −6 
We have, A =1( −3 − 0) − 0 + 0 =−3
|A |=−12 − ( −12) =−12 + 12 =0 Now,
1 0  0 0  A11 =−3 − 0 =−3, A12 =−( −3 − 0) =3, A13 =6 − 15 =−9
∴ A=I 0 =    A21 =−( 0 − 0) =
0, A 22 =−1 − 0 =−1, A 23 = −( 2 − 0) =−2
0 1  0 0 
A31 =0 − 0 =0, A32 =−( 0 − 0) =0, A 33 =3 − 0 =3
Now,
A11 =−6, A12 4, A21 =
= −3, A22 =
2  −3 0 0 
∴ adj. A =  3 −1 0 
 −6 −3
∴ adj. A =
4 2  −9 −2 3
 
 −3 0 0 
Now, 1 1
∴ A −1 = adj. A = −  3 −1 0 
 2 3   −6 −3 |A| 3
A( adj. A )=     −9 −2 3
[3]
 −4 −6   4 2 
Q. 11. Find the inverse of each of the matrices (if it exists)
 −12 + 12 −6 + 6 
=  2 1 3
 24 − 24 12 − 12  
4 −

1 0 .  [NCERT Ex. 4.5, Q. 9, Page 132]
0 0 

=   −7 2 1 
0 0  Ans. Let we assume that,
 −6 −3  2 3 
Also, ( adj. A ) A =     2 1 3
 4 2   −4 −6  =A  4 −1 0 
 −12 + 12 −18 + 18  −7 2 1 
=
 8−8 12 − 12  We have,
0 0  A = 2( −1 − 0) − 1( 4 − 0) + 3(8 − 7 )
= 
0 0  =2 ( −1) − 1( 4) + 3(1)
Hence, A( adj. A ) = ( adj. A ) A = A I = − 2 − 4+3
[3]
Q. 9. Find the inverse of each of the matrices (if it exists) =− 3
1 2 3 Now,
  A11 =−1 − 0 =−1, A12 =−( 4 − 0) =−4, A13 =8 − 7 =1
0 2 4  .
 0 0 5   [NCERT Ex. 4.5, Q. 7, Page 132] A21 =−(1 − 6) =5, A22 =2 + 21 =23, A23 =−( 4 + 7 ) =−11
Ans. Let we assume that, A31 =0 + 3 =3, A32 =−( 0 − 12) =12, A33 =−2 − 4 =−6
 1 2 3  −1 5 3
− 
A = 0 2 4  ∴ adj. A = 4 23 12 
0 0 5  1 −11 −6 
We have,  −1 5 3
1 1
A = 1(10 − 0) − 2(0 − 0) + 3(0 − 0)= 10 ∴ A −1 = adj. A = −  −4 23 12 
|A| 3
Now, [3]  1 −11 −6 
A11 =10 − 0 =10, A12 =−(0 − 0) =0, A13 =0 − 0 =0 Q. 12. Find the inverse of each of the matrices (if it exists)
A21 =−(10 − 0) = −10, A22 =−
5 0= 5, A23 = −(0 − 0) =0  1 −1 2 
A31 =8 − 6 =2 , A32 =−( 4 − 0) =−4 , A33 =2 − 0 =2  
 0 2 −3  .  [NCERT Ex. 4.5, Q. 10, Page 132]
10 −10 2   3 −2 4 
∴ adj. A =0 5 −4 
 Ans. Let we assume that,
 0 0 2 
1 −1 2 
10 −10 2 
1 1  =A 0 2 −3
∴=A −1 =
adj .A 0 5 −4 
|A| 10   3 −2 4 
[3]  0 0 2 
By expanding along C1 , we get
A = 1(8 − 6) − 0( −4 + 4) + 3(3 − 4)
=2 − 3
1 −1 2 
=A 0 2 −3
 3 −2 4  DETERMINANTS | 125
By expanding along C1 , we get This system of equations can be written in the form
AX = B, where
A = 1(8 − 6) − 0( −4 + 4) + 3(3 − 4)
1 1 1  x  1 
=2 − 3 
=A = 2 3 2  , X  y  and B=  2 
  
=− 1
 a a 2a   z   4 
Now,
Now,
A11 =8 − 6 =−2, A12 =−( 0 + 9) =−9, A13 =0 − 6 =−6
A = 1( 6a − 2a ) − 1( 4a − 2a ) + 1( 2a − 3a )
A21 =−( −4 + 4) =0, A22 =( 4 − 6) =−2, A23 =( −2 + 3) =1
= 4a − 2a − a = 4a − 3a = a ≠ 0
A31 = 3 − 4 = −1, A32 = −( −3 − 0) = 3, A33 = 2 − 0 =2
∴ A is non-singular.
 2 0 −1
  Therefore, A-1 exists.
∴ adj. A =−  9 −2 3  Hence, the given system of equations is consistent.
 −6 −1 2   [3]
 2 0 −1  −2 0 1  Q. 15. Examine the consistency of the system of
1
∴ A −1 = adj. A = −1  −9 −2 3  =
 9 2 −3
 equations.
|A| 3x − y − 2 z = 2
 −6 −1 2   6 1 −2 
[3] 2 y − z =−1
Q. 13. Find the inverse of each of the matrices (if it 3 x − 5y = 3  [NCERT Ex. 4.6, Q. 5, Page 136]
1 0 0  Ans. The given system of equations is :
exists)  0 cos α sin α  .

3x − y − 2 z = 2
 0 sin α − cos α  2 y − z =−1
 [NCERT Ex. 4.5, Q. 11, Page 132] 3x − 5 y = 3
Ans. Let we assume that, This system of equations can be written in the form
1 0 0  of AX = B, where
A = 0 cosα sin α   3 −1 −2  x 2
0 sin α − cosα  A 0 2 −1 ,=
= X  y  and B=  −1
We have,  3 −5 0   z   3 
A = 1( − cos2 α − sin 2 α ) Now,
| A|=3(0 − 5) − 0 + 3(1 + 4) =−15 + 15 =0
= − ( cos2 α + sin 2 α )
∴ A is a singular matrix.
=− 1 Now,
Now,  −5 10 5 
A11 = − cos2 α − sin 2 α = −1, A12 = 0, A13 = 0  
( adj. A )=  −3 6 3
A21 = 0, A22 = − cosα , A23 = − sin α  −6 12 6 
A31 = 0, A 32 = − sin α , A33 = cosα  −5 10 5   2   −10 − 10 + 15  −5
 −1 0 0  ∴ ( adj. A )B =  −3 6 3  −1 =  −6 − 6 + 9  =  −3 ≠ 0
∴ adj. A =  0 − cosα

− sin α   −6 12 6   3   −12 − 12 + 18  −6 
 0 − sin α cosα  Thus, the solution of the given system of equations
does not exist. Hence, the system of equations is
 −1 0 0 
1  inconsistent. [3]
∴A = −1
.adj. A = −1  0 − cosα − sin α  Q. 16. Examine the consistency of the system of
A
 0 − sin α cosα  equations.
1 0 0  5x − y + 4 z = 5
= 0 cosα s in α  2 x+3 y + 5 z = 2
0 sin α − cosα  5x − 2y + 6 z = −1
 [3]
 [NCERT Ex. 4.6, Q. 6, Page 136]
Q. 14. Examine the consistency of the system of
Ans. The given system of equations is :
equations.
5x − y + 4 z =5
x+y+ z= 1
2x+3y + 5z = 2
2 x + 3y + 2 z = 2
5x − 2 y + 6 z =
−1
ax + ay + 2 az = 4
 [NCERT Ex. 4.6, Q. 4, Page 136] This system of equations can be written in the form
of AX = B, where
Ans. The given system of equations is :
x+y+z= 1  5 −1 4  x  5
= 
A 2 =  
3 5  , X = y  and B  2 

2x + 3y + 2 z =2
ax + ay + 2az = 4  5 −2 6   z  1 
Now,
A= 5(18 + 10) + 1(12 − 25) + 4( −4 − 15)
= 5( 28) + 1( −13) + 4( −19)
 5 −1 4  x  5
A =  2 3 5  , X =  y  and B =  2 
   
126 | OSWAAL NCERT SOLUTIONS – Textbook + Exemplar – MATHEMATICS : Class-XII
 5 −2 6   z  1 
Now, Q. 19. Solve system of linear equations, using matrix
A = 5(18 + 10) + 1(12 − 25) + 4( −4 − 15) method.
4 x − 3y = 3
= 5( 28) + 1( −13) + 4( −19) 
3 x − 5y = 7 [NCERT Ex. 4.6, Q. 9, Page 136]
= 140 − 13 − 76
Ans. The given system of equations can be written in the
= 51 ≠ 0 form of AX = B, where
∴ A is non-singular.
 4 −3 x  3
Therefore, A–1 exists. A=  , X =  y  and B = 7 
Hence, the given system of equations is consistent.  3 −5     
 [3] Now,
Q. 17. Solve system of linear equations, using matrix A = −20 + 9 = −11 ≠ 0
method.
5x + 2y = 4  Thus, A is non-singular. Therefore, its inverse
exists.
7 x + 3y = 5 [NCERT Ex. 4.6, Q. 7, Page 136] Now,
Ans. The given system of equations can be written in the
1 1  −5 3  1 5 −3
form of AX = B, where A −1 = ( adj. A ) = −  =  
| A| 11  −3 4  11 3 −4 
5 2 x  4
A=  , X =   and B=   1 5 −3  3 
7 3  y 5 ∴ X = A −1B =   
11 3 −4  7 
Now, A = 15 − 14 = 1 ≠ 0
 −6 
Thus, A is non-singular. Therefore, its inverse  x  1 5 −3  3  1 15 − 21 1  −6   11 
exists. ⇒ =    =  =  = 
Now,  y  11 3 −4  7  11  9 − 28  11  −19   −19 
 11 
1 −6 −19
A −1 = ( adj. A) Hence, x = and y = .
| A| [3] 11 11
 3 −2  x sin θ cos θ
∴ A −1 =  
 −7 5  Q. 20. Prove that the determinant − sin θ − x 1 is
 3 −2   4  cos θ 1 x
∴ X = A −1B =    independent of θ.
 −7 5   5   [NCERT Misc. Ex. Q. 1, Page 141]
 x   12 − 10   2  Ans. We know that,
⇒   = − +  = − 
 y   28 25  3 x sin q cosq
Hence, x = 2 and y = −3 [3] ∆ = − sin q −x 1
Q. 18. Solve system of linear equations, using matrix cosq 1 x
method. = x( − x 2 − 1) − sin q ( − x sin q − cosq ) + cosq ( − sin q + x cosq )
2 x − y = −2
 = − x 3 − x + x sin 2 q + sin q cosq − sin q cosq + x cos2 q
3x + 4y = 3 [NCERT Ex. 4.6, Q. 8, Page 136]
= − x 3 − x + x(sin 2 q + cos2 q )
Ans. The given system of equations can be written in the
= −x 3 − x + x
form of AX = B, where
= − x 3 (Indenpendent of q )
 2 −1 x   −2 
A=  , X =  y  and B=  3  Hence, Δ is independent of θ. [3]
 3 4      Q. 21. Without expanding the determinant, prove that
Now,
a a2 bc 1 a 2 a3
|A |=8+3=11 ≠ 0
b b2 ca = 1 b2 b3 .
Thus, A is non-singular. Therefore, its inverse
exists. c c 2 ab 1 c 2 c 3
Now,  [NCERT Misc. Ex. Q. 2, Page 141]
1 1  4 1 Ans. Taking LHS, we get
A −1 = adj. A =  
|A 11  −3 2 
a a2 bc
1  4 1   −2 
∴ X = A −1B =    LHS = b b 2 ca
11  −3 2   3 
c c2 ab
 −5 
 x  1  −8 + 3 1  −5  11  a2 a3 abc
⇒  =  =  =  1 2
 y  11  6 + 6  11 12   12  = b b3 abc
 11  abc 2
c c3 abc
−5 12
Hence, x = and y = [ R1 → aR1 , R2 → bR2 , and R3 → cR3 ]
11 11  [3]
a2 a3 1
1
= .abc b 2 b3 1
abc
c2 c3 1
c c ab α α β +γ
a 2
a 3
abc ∆ =( β − α )(γ − α ) 1 β + α −1
1 2 0 γ −β 0
= b b3 abc
abc 2
c c3 abc Expanding along R3 , we have
DETERMINANTS
∆ =( β − α )(γ − α )[ −(γ − β )( −α − β − γ )]
| 127
[ R1 → aR1 , R2 → bR2 , and R3 → cR3 ]
a2 a3 1 =–( β − α )(γ − α )( β − γ )(α + β + γ )]
1 = (α − β )( β − γ )(γ − α )(α + β + γ )
= .abc b 2 b3 1
abc Hence proved. [3]
c2 c3 1
Q. 24. Using properties of determinants, prove that
[Taking out factor abc from C3 ] 1 1+ p 1+ p+ q
a2 a3 1 2 3 + 2 p 4 + 3 p + 2q = 1.
= b2 b3 1 3 6 + 3 p 10 + 6 p + 3 q
c2 c3 1  [NCERT Misc. Ex. Q. 14, Page 142]
Ans. Taking LHS,
1 a2 a3
1 1+ p 1+ p + q
= 1 b2 b3
= ∆ 2 3 + 2 p 4 + 3p + 2q
1 c2 c3
3 6 + 3p 10 + 6 p + 3q
[Applying C1 ↔ C3 and C2 ↔ C3 ] Applying R2 → R2 − 2R1 and R3 → R3 − 3R1 , we have
Hence proved. [3] 1 1+ p 1+ p + q
cos α cos β cos α sin β − sin α
∆= 0 1 2+p
Q. 22. Evaluate − sin β cos β 0
0 3 7 + 3p
sin α cos β sin α sin β cos α
Applying R3 → R3 − 3R2 , we have
 [NCERT Misc. Ex. Q. 3, Page 141]
1 1+ p 1+ p + q
Ans. Given that,
∆= 0 1 2+p
cosα cos β cosα sin β − sin α
0 0 1
∆= − sin β cos β 0
Expanding along C1, we have
sin α cos β sin α sin β cosα
1 2+p
Expanding along C3, we have ∆= 1 = 1(1 − 0=
) 1
0 1
∆ = − sin α ( − sin α sin 2 β − cos2 β sin α )
Hence proved. [3]
+ cosα (cosα cos2 β + cosα sin 2 β ) Q. 25. Using properties of determinants, prove that
= sin 2 α (sin 2 β + cos2 β ) sin α cosα cos(α + δ )
+ cos2 α (cos2 β + sin 2 β ) sin β cos β
cos( β + δ ) =
0
= sin α (1) + cos α (1)
2 2 sin γ cos γ
cos(γ + δ )
=1 [3]  [NCERT Misc. Ex. Q. 15, Page 142]
Q. 23. Using properties of determinants, prove that Ans. We are taking LHS
sin α cos α cos(α + δ )
α α2 β + γ
∆ = sin β cos β cos( β + δ )
β β 2 γ + α = (β − γ )( γ − α)(α − β )(α + β + γ ).
sin γ cos γ cos(γ + δ )
γ γ2 α + β
sin α sin δ cos α cos δ cos α cos δ − sin α sin δ
 [NCERT Misc. Ex. Q. 11, Page 142] 1
= sin β sin δ cos β cos δ cos β cos δ − sin β sin δ
Ans. Let we assume that, sin δ cos δ
sin γ sin δ cos γ cos δ cos γ cos δ − sin γ sin δ
α α2 β +γ
Applying C1 → C1 + C3 , we have
= ∆ β β2 γ +α
cos α cos δ cos α cos δ cos α cos δ − sin α sin δ
γ γ2 α +β 1
∆= cos β cos δ cos β cos δ cos β cos δ − sin β sin δ
Applying R2 → R2 − R1 and R3 → R3 − R1 , we have sin δ cos δ
cos γ cos δ cos γ cos δ cos γ cos δ − sin γ sin δ
α α2 β +γ Here, two columns C1 and C 2 are identical.
∆= β − α β −α2 α − β
2
∴ ∆=0
γ −α γ 2 −α2 α −γ Hence proved. [3]
α2 α β +γ Q. 26. Using the properties of determinants, evaluate
3x −x + y −x + z
=( β − α )(γ − α ) 1 β + α −1
x−y 3y z−y
1 γ +α −1
x−z
y− z 3z
Applying R3 → R3 − R2 , we have
 [NCERT Exemp. Ex. 4.3, Q. 4, Page 77]
α2 α β +γ Ans. We have,
∆ =( β − α )(γ − α ) 1 β + α −1 3x −x + y −x + z
0 γ −β 0 x−y 3y z−y
Expanding along R3 , we have x−z y−z 3z
∆ =( β − α )(γ − α )[ −(γ − β )( −α − β − γ )] Applying C1 → C1 + C2 + C3 ,
=–( β − α )(γ − α )( β − γ )(α + β + γ )] x + y + z −x + y −x + z
= (α − β )( β − γ )(γ − α )(α + β + γ ) = x+y+z 3y z−y
x+y+z y−z 3z
3x −x + y −x + z y2z2 yz y+z
x−y 3y z−y 2
zx 2
zx z + x
x−z y−z 3z x2y2 xy x + y
128 | OSWAAL NCERT SOLUTIONS – Textbook + Exemplar – MATHEMATICS : Class-XII
Applying C1 → C1 + C2 + C3 , [Multiplying R1 , R2 , R3 by x , y , z respectively]
x + y + z −x + y −x + z xy 2 z 2 xyz xy + xz
1 2 2
= x+y+z 3y z−y = x yz xyz yz + xy
xyz 2 2
x+y+z y−z 3z x y z xyz xz + yz
1 −x + y −x + z [Taking (xyz ) common from C1 and C2 ]
= (x + y + z) 1 3y z−y yz 1 xy + xz
1 2
1 y−z 3z = ( xyz ) xz 1 yz + xy
xyz
[Taking ( x + y + z ) common from column C ] xy 1 xz + yz
l
Applying C3 → C3 + C1
1 −x + y −x + z
yz 1 xy + xz + zx
= (x + y + z) 0 2y + x x − y
= xyz xz 1 yz + xy + zx
0 x − z 2z + x
xy 1 xz + yz + zx
[ R2 → R2 − R1 and R3 → R3 − R1 ]
[Taking (xy + yz + zx ) common from C3 ]
Now, expanding along first column, we get
yz 1 1
= ( x + y + z ).1[( 2 y + x )( 2 z + x ) − ( x − y )( x − z )]
= xyz( xy + yz + zx ) xz 1 1
= ( x + y + z )( 4 yz + 2 yx + 2xz + x 2 − x 2 + xz + yx − yz ) xy 1 1
= ( x + y + z )(3yz + 3yx + 3xz ) =0 [  C2 and C3 are identical]
 [3]
= 3( x + y + z )( yz + yx + xz )
[3] Q. 29. Using the properties of determinants, prove that
Q. 27. Using the properties of determinants, evaluate y+ z z y
a−b− c 2a 2a z z+ x x =4 xyz.
2b b− c− a 2b . y x x+y
2c 2c c−a−b  [NCERT Exemp. Ex. 4.3, Q. 8, Page 77]
 [NCERT Exemp. Ex. 4.3, Q. 6, Page 77] Ans. y+z z y
Ans. We have,
z z+x x
a−b−c 2a 2a
y x x+y
2b b−c−a 2b
[Applying C1 → C1 + C2 + C3 ]
2c 2c c−a−b
2( y + z ) z y
Applying R1 → R1 + R2 + R3
=2( z + x ) z + x x
a+b+c a+b+c a+b+c
2( y + x ) x x+y
= 2b b−c−a 2b
y+z z y
2c 2c c−a−b
=2 z + x z + x x
[Taking (a + b + c ) common from the first row]
y+x x x+y
1 1 1
[Applying C1 → C1 − C2 ]
= ( a + b + c ) 2b b − c − a 2b
y z y
2c 2c c−a−b
= 2 0 z+x x
[Applying C1 → C1 − C3 and C2 → C2 − C3 ]
y x x+y
0 0 1
[Applying C3 → C3 − C1 ]
= (a + b + c) 0 −( a + b + c ) 2b
y z 0
a+b+c a+b+c c−b−a
= 2 0 z+x x
Expanding along R1 , y x x
= ( a + b + c )[1 × 0 + ( a + b + c )2 ] [Applying R3 → R3 − R1 ]
= ( a + b + c )3 [3] z y0
Q. 28. Using the properties of determinants, prove that = 2 0 z+x x
y 2 z 2 yz y + z 0 x−z x
z 2 x 2 zx z + x =0 = 2 y[( z + x )x − x( x − z )]
x 2 y 2 xy x + y = 2 y( 2xz )
 [NCERT Exemp. Ex. 4.3, Q. 7, Page 77] = 4xyz [3]
Ans. y2z2 yz y + z Q. 30. If A + B + C = 0, then prove that
z 2x 2 zx z + x 1 cos C cos B
x2y2 xy x + y cos C 1 cos A = 0
cos B cos A 1
[Multiplying R1 , R2 , R3 by x , y , z respectively]
 [NCERT Exemp. Ex. 4.3, Q. 10, Page 78]
xy 2 z 2 xyz xy + xz
1 2 2
= x yz xyz yz + xy
xyz 2 2
x y z xyz xz + yz
DETERMINANTS | 129

1 cos C
cos B Ans. We have,
Ans.
cos C cos A =
1 1(1 − cos2 A ) − cos C(cos C 11 11 sin sin 33qq
cos B cos A 1 −44 33 cos
− cos 22qq == 00
− cos A.cos B) + cos B(cos C.cos A − cos B) 77 − −77 −
−22
= sin 2 A − cos2 C + cos A.cos Expanding along
Expanding along C C3 ,, we
we getget
3

B.cos C + cos A.cos B.cos C − cos2 B sin q × − −


sin 3q × ( 28 − 21) − cos 2q × (( −
3 ( 28 21 ) cos 2 q × −77 −
− 77 )) −
− 22((33 +
+ 44)) =
=
00
⇒ 7 sin 3 q + 14 cos 2
⇒ 7 sin 3q + 14 cos 2q − 14 = q − 14 =0
0
= sin 2 A − cos2 B + 2 cos A.cos
⇒ sin 33qq +
⇒ sin cos 22qq −
+ 22 cos − 22 =
= 00
B.cos C − cos2 C
⇒ sin qq −
⇒ ((33 sin sin 3 qq )) +
− 44 sin + 22((11 − sin 2 qq )) −
− 22 sin − 22 =
3 2
=00
= − cos( A + B).cos( A − B) + 2 cos
⇒ sin 3 qq −
⇒ 44 sin sin 2 qq +
− 44 sin sin qq =
+ 33 sin
3 2
= 00
A.cos B.cos C − cos2 C
⇒ sin qq (( 44 sin
⇒ sin sin 22 qq − sin qq +
− 44 sin + 33)) =
00
=
[ cos2 B − sin 2 A =cos(A + B).cos(A − B)]
= − cos( −C ).cos(A − B) ⇒ sin θ ( 4 sin 2 θ − 6 sin θ + 2 sin θ + 3) = 0
+ cosC( 2 cosA.cosB − cosC) ⇒ sin θ ( 2 sin θ + 1)( 2 sin θ − 3) = 0
= − cosC(cosA.cosB + sinA.sinB 1 3
⇒ sin θ = 0 or sinθ = − or sinθ =
2 2
− 2 cosA.cosB+ cosC)
 π 
= cosC(cosA.cosB − sinA.sinB − cosC) ⇒ θ = nπ or θ =mπ +( − 1)n  −  ; m, n ∈ Z
 6
= cosC[cos(A + B) − cosC] 3
= cosC(cosC − cosC) ⇒ sin θ = − is not possible.
2  [3]
[ As cosC=cos(A+B)]
4 - x 4+ x 4+ x
=0  [3] Q. 33. Find the value of x if, 4 + x 4 - x 4 + x = 0
Q. 31. If the co-ordinates of the vertices of an equilateral
4+ x 4+ x 4 - x
triangle with sides of length ‘a’ are (x1, y1), (x2, y2),
2
x1 y1 1  [NCERT Exemp. Ex. 4.3, Q. 13, Page 78]
3a4
(x3, y3),, then x2 y2 1 = . Ans. We have,
4
x3 y3 1 4−x 4+x 4+x
 [NCERT Exemp. Ex. 4.3, Q. 11, Page 78] 4+x 4−x 4+x =
0
Ans. The area of a triangle with vertices (x1, y1), (x2, y2),
4+x 4+x 4−x
and (x3, y3), is given by
x1 y1 1 [Applying R1 → R1 + R2 + R3 ]
1 12 + x 12 + x 12 + x
∆ = x2 y2 1
2 ⇒ 4+x 4−x 4+x = 0
x3 y3 1
Also area of an equilateral triangle with side 4+x 4+x 4−x
a is given by [Taking (12 + x ) common from R1 ]
1 1 1
3 2
∆= a ⇒ (12 + x ) 4 + x 4 − x 4 + x =
0
4
x1 y1 1 4+x 4+x 4−x
1 3 2 [Applying C1 → C1 − C3 and C2 → C2 − C3 ]
∴ x2 y2 1 = a
2 4
x3 y3 1 0 0 1
Squaring both sides, we get ⇒ (12 + x ) 0 −2x 4 + x =
0
x1 y1 1
2 2x 2x 4−x
1 3 4 ⇒ (12 + x )[0 − ( −2x )( 2x )] =0
=⇒∆ =x2 y2 1 2
a
4 16 ⇒ (12 + x )( 4x 2 ) =
0
x3 y3 1
2 ∴ x =−12, 0  [3]
x1 y1 1
3a 4 Q. 34. Using matrix method, solve the system of
or x2 y2 1 =
4 equations
x3 y3 1  [3]
3x + 2y − 2 z =
3,
1 1 sin 3θ
x + 2y + 3z =6
Q. 32. Find the value of θ satisfying −4 3 cos 2θ =0
2x − y + z =2.
7 −7 −2
 [NCERT Exemp. Ex. 4.3, Q. 19, Page 79]
 [NCERT Exemp. Ex. 4.3, Q. 12, Page 78]
130 | OSWAAL NCERT SOLUTIONS – Textbook + Exemplar – MATHEMATICS : Class-XII

Ans. Given that system of equations is = 9[( − y )( − z ) − 0 + x( y + 3yz + z )]


3x + 2 y − 2 z =
3,
= 9[ yz + xy + 3xyz + xz]
x + 2 y + 3z =6
= 9(3xyz + xy + yz + zx )
2x − y + z = 2.
Hence proved. [4]
In the form of AX = B,
Q. 36. Using properties of determinants, prove that
3 2 −2   x  3 
1 2 3   y  = 6  x x+y x + 2y
    
 2 −1 1   z   2  x + 2y x x + y = 9 y 2 ( x + y)
x+y x + 2y x
For A −1 .
 [CBSE Board, Delhi Region, 2017]
A = 3(5) − 2(1 − 6) + ( −2)( −5)
Ans.
= 15 + 10 + 10
x x + y x + 2y
= 35 ≠ 0 x + 2y x x+y
∴ A11 = 5, A12 = 5, A13 = −5, A21 = 0, A22 = 7 , A23 x + y x + 2y x
= 7 , A31 = 10, A32 = −11 and A33 = 4 C1 → C1 + C2 + C3
5 5 −5 5 0 10 1 x + y x + 2y
∴ adj. A = 0 7 7 = 5 7 −11 =
3( x + y ) 1 x x+y
10 −11 4 −5 7 4 1 x + 2y x
5 0 10 R1 → R1 − R2 , R3 → R3 − R2
adj. A 1
Now, A −1 = = 5 7 −11 0 y y
A 35
−5 7 4 =
3( x + y ) 1 x x+y
0 2y − y
For X = A −1B
=−3(x + y)( − y 2 − 2 y 2 )
x  5 0 10  3 
 y  = 1 5 7 −11 6  = 9y 2 (x + y)
  35    [4]
 z   −5 7 4   2  Q. 37. Using properties of determinants, prove that
15 + 20 
1  a2 + 2 a 2 a + 1 1
= 15 + 42 − 22 
35 2a + 1 a + 2 1 = ( a − 1)3
 −15 + 42 + 8
3 3 1
35 1 .
1     [CBSE Board, All India Region, 2017].
= 35 = 1
35     Ans.
35 1
∴ x = 1, y = 1, z = 1 a 2 + 2a 2a + 1 1
[3]
Q. 35. Using properties of determinates, prove that ∆
= 2a + 1 a + 2 1
1 1 1 + 3x 3 3 1
1 + 3y 1 1 = 9(3 xyz + xy + yz + zx) R1 → R1 − R2 and R2 → R2 − R3
1 1 + 3z 1 a2 − 1 a −1 0
.
 [CBSE Board, Delhi Region, 2018] =
∆ 2( a − 1) a − 1 0
Ans. Given that, 3 3 1
1 1 1 + 3x a +1 1 0
1 + 3y 1 1 = ( a − 1)2 2 1 0
1 1 + 3z 1 3 3 1
Apply R1 → R1 − R2 and R2 → R2 − R3 On expanding, we get
1 1 1 + 3x −3y 0 3x =−
( a 1)2 .( a − 1)
1 + 3y 1 1 =3y −3z 0 = ( a − 1)3 [4]
1 1 + 3z 1 1 1 + 3z 1
Taking 3 common from R1 and R2 , we get
−3y 0 3x −y 0 x
3y −3z 0 = (3)(3) y −z 0
1 1 + 3z 1 1 1 + 3z 1 
DETERMINANTS | 131

Long Answer Type Questions (5 or 6 marks each)

Q. 1. Using the property of determinants and without a p x


expanding, prove that ∆=2b q y
b+ c q+ r y+ z a p x
c r z
c+a r+ p z+ x =2 b q y
Hence proved. [5]
a+b p+ q x+ y c r z
Q. 2. By using properties of determinants, show that :
 [NCERT Ex. 4.2, Q. 5, Page 119] 1 a a2
Ans. (i) 1 b b2 = ( a − b)( b − c )( c − a)
b+c q+r y+z
1 c c2
∆= c+a r+p z+x
a+b p+q x+y 1 1 1
b+c q+r y+z b+c q+r y+z (ii) a b c = ( a − b)( b − c )( c − a)( a + b + c )
= c+a r+p z+x + c+a r+p z+x a3 b3 c3
a p x b q y  [NCERT Ex. 4.2, Q. 8, Page 120]
= ∆1 + ∆ 2 (say)  …(i) Ans.
(i) Let we assume that
b+c q+r y+z
1 a a2
Now, ∆1 = c + a r + p z + x
∆ = 1 b b2
a p x
1 c c2
Applying R2 → R2 – R3, we have
b+c q+r y+z Applying R1→R1–R3 and R2→R2–R3, we have
∆1 = c r z 0 a − c a2 − c 2
a p x ∆ = 0 b − c b2 − c2
Applying R1 → R1 − R2 , we get 1 c c2

b q y 0 −1 −a − c
∆1 = c r z = ( b − c )( c − a ) 0 1 b+c
a p x 1 c c2

Applying R1↔R3 and R2↔R3 we have Applying R1→R1+R2 we have


a p x a p x 0 0 −a + b
∆1 = ( −1)2 b q y = b q y   …(ii) ∆ = ( b − c )( c − a ) 0 1 b + c
c r z c r z 1 c c2
b+c q+r y+z 0 0 −1
∆2 = c + a r + p z + x = ( a − b )( b − c )( c − a ) 0 1 b + c
b q y 1 c c2
Expanding along C1, we have
Applying R1→R1–R3, we have
c r z 0 −1
∆ = ( a − b )( b − c )( c − a )
∆2 = c + a r + p z + x 1 b+c
b q y = ( a − b )( b − c )( c − a )
Applying R2 → R2 − R1 , we get Hence proved. [2½]
(ii) Let we assume that
c r z
1 1 1
∆2 = a p x
∆= a b c
b q y
a3 b 3 c 3
Applying R1↔R2 and R2↔R3 we have
Applying C1→C1−C3 and C2→C2−C3 we have
a p x a p x
0 0 1
∆ 2 = ( −1)2 b q y = b q y ...(iii)
∆= a−c b−c c
c r z c r z
a3 – c 3 b – c c3
3 3

From (i), (ii) and (iii), we have


0 0 1
= a−c b−c c
( a − c )( a 2 + ac + c 2 ) (b − c )(b 2 + bc + c 2 ) c 3
0 0 1
0 0 1
132 | ∆ = a − c b − c
OSWAAL NCERT SOLUTIONSc – Textbook + Exemplar – MATHEMATICS : Class-XII
a – c b – c c3
3 3 3 3

0 0 1  x yz x x2 
∆ = [( x − y )( z − x )( z − y )] ( −1) +1 
= a−c b−c c −1 z −1 − x − y
 
( a − c )( a 2 + ac + c 2 ) (b − c )(b 2 + bc + c 2 ) c 3
= ( x − y )( z − x )( z − y ) ( − xz − yz ) + ( − x 2 − xy + x 2 )
0 0 1
= −( x − y )( z − x )( z − y )( xy + yz + zx )
= ( c − a)(b − c ) −1 1 c
= ( x − y )( y − z )( z − x )( xy + yz + zx )
−( a + ac + c ) (b + bc + c ) c 3
2 2 2 2
                            
Hence proved. [5]
Applying C1→C1+C2 we have
Q. 4. By using properties of determinants, show that :

0 0 1 (i) x + 4 2 x 2x
2x x + 4 2 x = (5 x + 4)(4 − x )2
∆ = ( c − a )( b − c ) 0 1 c
2x 2x x+4
( b − a ) + ( bc − ac ) ( b + bc + c ) c 3
2 2 2 2

(ii) y + k y y
0 0 1
y y+k y = k 2 (3 y + k )
= ( b − c )( c − a )( a − b ) 0 1 c
y y y+k
−( a + b + c ) ( b 2 + bc + c 2 ) c 3
 [NCERT Ex. 4.2, Q. 10, Page 120]
0 0 1 Ans.
= ( a − b )( b − c )( c − a )( a + b + c ) 0 1 c (i) Let we assume that,
−1 ( b 2 + bc + c 2 ) c 3 x + 4 2x 2x
Expanding along C1, we have ∆ = 2x x + 4 2x
0 1 2x 2x x+4
∆ = (a− b)( b − c )( c − a )( a + b + c )( −1)
1 c
Applying R1→R1+R2+R3 we have
= ( a − b )( b − c )( c − a )( a + b + c ) 5x + 4 5x + 4 5x + 4
Hence proved. [2½]
∆ = 2x x + 4 2x
Q. 3. By using properties of determinants, show that :
2x 2x x+4
x x 2 yz
1 1 1
y y 2 zx = ( x − y )( y − z )( z − x )( xy + yz + zx )
= ( 5x + 4 ) 2 x x + 4 2 x
z z 2 xy
 [NCERT Ex. 4.2, Q. 9, Page 120] 2x 2x x+4
Ans. Let we assume that, Applying C2→C2−C1 and C3→C3−C1 we have
x x2 yz 1 0 0
∆ = y y2 zx ∆ = ( 5x + 4 ) 2 x − x + 4 0
z z2 xy 2x 0 −x + 4
Applying R2→R2−R1 and R3→R3−R1 we have 1 0 0
x x2 yz = (5x + 4)( 4 − x )( 4 − x ) 2x 1 0
∆ = y − x y2 − x2 zx − yz 2x 0 1
z − x z2 − x2 xy − yz Expanding along C3 we have
x x2 yz 1 0
∆ = (5x + 4)( 4 − x )2
= −( x − y ) −( x − y )( x + y ) z( x − y ) 2x 1
(z − x) ( z − x )( z + x ) − y( z − x ) = (5x + 4)( 4 − x )2
Hence proved. [2½]
x x2 yz (ii) Let we assume that
= ( x − y )( z − x ) −1 − x − y z y+k y y
1 z+x −y ∆= y y+k y
Applying R3→R3+R2 we have y y y+k
x x2 yz Applying R1→R1+R2+R3 we have
∆ = ( x − y )( z − x ) −1 − x − y z 3y + k 3y + k 3y + k
0 z−y z−y ∆= y y+k y
x x2 yz y y y+k
= ( x − y )( z − x )( z − y ) −1 − x − y z 1 1 1
0 1 1 = ( 3y + k ) y y + k y
Expanding along R3 we have y y y+k
DETERMINANTS | 133
Applying C2→C2−C1, C3→C3−C1 we have 1 x y
1 0 0 = 2( x + y + z ) 1 y + z + 2x y
∆ = ( 3y + k ) y k 0 1 x z + x + 2y
y 0 k
Applying R2 → R2 – R1 and R3 → R3 – R1, we have
1 0 0
1 x y
= k ( 3y + k ) y 1 0
2

∆ = 2( x + y + z ) 0 x + y + z 0
y 0 1
0 0 x+y+z
Expanding along C3 we have
1 0 1 x y
∆ = k 2 ( 3y + k ) = 2( x + y + z ) 0 1 0
3
y 1
0 0 1
=k 2 (3y + k )
Hence proved. [2½]
Q. 5. By using properties of determinants, show that : Expanding along R3, we have
a − b − c 2a 2a ∆ = 2( x + y + z )3 (1)(1 − 0) = 2( x + y + z )3
(i) 2 b b − c − a 2b = ( a + b + c )3
2c 2c c−a−b Hence proved. [2½]
Q. 6. By using properties of determinants, show that :
x + y + 2z x y
(ii) z y + z + 2x y = 2( x + y + z )3 1 x x2
z x z + x + 2y
2
x 1 x = (1 − x 3 )2
2
x x 1
 [NCERT Ex. 4.2, Q. 11, Page 120]
Ans. (i) Let we assume that,  [NCERT Ex. 4.2, Q. 12, Page 121]
a − b − c 2a 2a Ans. Let we assume that,
∆ = 2b b − c − a 2b 1 x x2
2c 2c c−a−b ∆ = x2 1 x
Applying R1→R1+R2+R3, we have x x2 1
a+b+c a+b+c a+b+c Applying R1 → R1 + R2 + R3, we have
∆ = 2b b − c − a 2b 1 + x + x2 1 + x + x2 1 + x + x2
2c 2c c−a−b ∆ = x2 1 x
1 1 1 x x 2
1
=(a + b + c ) 2b b − c − a 2b 1 1 1
2c 2c c−a−b = (1 + x + x 2 ) x 2 1 x
Applying C2→C2–C1, C3→C3–C1, we have x x2 1
1 0 0 Applying C2 → C2 – C1 and C3 → C3 – C1, we have
∆ = ( a + b + c ) 2b −( a + b + c ) 0 1 0 0
2c 0 −( a + b + c ) ∆ = (1 + x + x 2 ) x 2 1 − x 2 x − x 2
1 0 0 x x2 − x 1 − x
= ( a + b + c ) 2b −1 0
3
1 0 0
2c 0 −1 = (1 + x + x 2 )(1 − x )(1 − x ) x 2 1 + x x
Expanding along C3, we have
x −x 1
∆ = ( a + b + c )3 ( −1)( −1)
1 0 0
=( a + b + c )3
= (1 − x 3 )(1 − x ) x 2 1 + x x
Hence proved. [2½] x −x 1
(ii) Let we assume that,
Expanding along R1, we have
x + y + 2z x y
1+ x x
∆= z y + z + 2x y ∆ = (1 − x 3 )(1 − x )(1)
−x 1
z x z + x + 2y
Applying C1 → C1 + C2 + C3, we have = (1 − x 3 )(1 − x )(1 + x + x 2 )
2( x + y + z ) x y = (1 − x 3 )(1 − x 3 )
∆ = 2( x + y + z ) y + z + 2x y = (1 − x 3 )2
2( x + y + z ) x z + x + 2y Hence proved. [5]
134 | OSWAAL NCERT SOLUTIONS – Textbook + Exemplar – MATHEMATICS : Class-XII

Q. 7. By using properties of determinants, show that : a2 + 1 b2 c 2


1 + a − b 2 ab
2 2
−2 b 1
∆ = abc × −1 1 0
2 ab 1 − a 2 + b2 2 a = (1 + a2 + b2 )3 abc
−1 0 1
2b −2 a 1 − a 2 − b2
a2 + 1 b2 c 2
 [NCERT Ex. 4.2, Q. 13, Page 121] = −1 1 0
Ans. Let we assume that, −1 0 1
1 + a 2 − b 2 2ab −2b
Expanding along R3, we get
∆ = 2ab 1 − a 2 + b 2 2a
b2 c2 a2 + 1 b2
2b −2a 1 − a2 − b2 ∆ = ( −1) +1
1 0 −1 1
Applying R1 → R1 + bR3 and R2 → R2 – aR3, we have = −1( − c 2 ) + ( a 2 + 1 + b 2 )
1 + a2 + b2 0 −b(1 + a 2 + b 2 ) = 1 + a2 + b2 + c 2
∆= 0 1 + a + b a(1 + a 2 + b 2 )
2 2
Hence proved. [5]
2b −2a 1 − a2 − b2 Q. 9. Show that points A ( a , b + c ) , B ( b , c + a ) , C ( c , a+b )
1 0 −b are collinear.
 [NCERT Ex. 4.3, Q. 2, Page 123]
= (1 + a + b ) 0
2 2 2
1 a
Ans. Area of ∆ABC is given by the relation,
2b −2a 1 − a 2 − b 2 a b+c 1
1
Expanding along R1, we have ∆= b c+a 1
2
 1 a 0 1  c a+b 1
∆ = (1 + a 2 + b 2 )2 (1) −b 
 − 2 a 1 − a 2
− b 2
2b −2a  a b+c 1
1
= (1 + a 2 + b 2 )2 1 − a 2 − b 2 + 2a 2 − b( −2b ) ∆ = b−a a−b 0
2
c−a a−c 0
= (1 + a 2 + b 2 )2 (1 + a 2 + b 2 )
[Applying R2 → R2 − R1 and R3 → R3 − R1 ]
= (1 + a 2 + b 2 )3
a b+c 1
Hence proved. [5] 1
= ( a − b )( c − a ) −1 1 0
Q. 8. By using properties of determinants, show that : 2
1 −1 0
a2 + 1 ab ac
a b+c 1
ab b + 1 bc
2
= 1 + a 2 + b2 + c 2 1
= ( a − b )( c − a ) −1 1 0
ca cb c +1
2
2
0 0 0
 [NCERT Ex. 4.2, Q. 14, Page 121]
[Applying R3 → R3 + R2 ]
Ans. Let we assume that,
=0 [All elements of R3 are 0]
a 2 + 1 ab ac
Thus, the area of the triangle formed by points A,
∆ = ab b 2 + 1 bc
B and C is zero. Hence, the points A, B and C are
ca cb c2 + 1 collinear.
Taking out common factors a, b and c from R1, R2 Q. 10. Find values of k if area of triangle is 4 sq. units and
and R3 respectively, we have vertices are
(i) ( k , 0 ) , ( 4, 0 ) , ( 0, 2 )
1
a+ b c
a (ii) ( –2, 0 ) , ( 0, 4 ) , ( 0, k )
1  [NCERT Ex. 4.3, Q. 3, Page 123]
∆ = abc a b+ c
b Ans. We know that the area of a triangle whose vertices
1 are ( x1 , y1 ) , ( x 2 , y 2 ) and ( x3 , y3 ) is the absolute
a b c+ value of the determinant (∆), where
c
x1 y1 1
Applying R2 → R2 – R1 and R3 → R3 – R1, we have 1
∆ = x 2 y2 1
1 2
a+ b c x3 y3 1
a
1 1 It is given that the area of triangle is 4 square units.
∆ = abc − 0 Therefore, Δ=±4. [1]
a b
1 1 (i) The area of the triangle with vertices (k, 0), (4, 0)
− 0 and (0, 2) is given by the relation
a c
k 0 1
Applying C1 → aC1, C2 → bC2, and C3 → cC3, we 1
∆= 4 0 1
have 2
0 2 1
DETERMINANTS | 135
When − k + 4 = − 4, k = 8. A22 = Co-factor of a22 = ( −1 ) M22 = 1
2 + 2

When − k + 4 = 4, k = 0.
A23 = Co-factor of a23 = ( −1 )
2 + 3
M23 = 0
Hence, k = 0, 8 [2]
A31 = Co-factor of a31 = ( −1 )
3 +1
M31 = 0
(ii) The area of the triangle with vertices (−2, 0), (0, 4)
A32 = Co-factor of a32 = ( −1 )
3 + 2
and (0, k) is given by the relation M32 = 0
−2 0 1
A33 = Co-factor of a33 = ( −1 )
3 + 3
1 M33 = 1
∆= 0 4 1 [5]
2 1 0 4
0 k 1
(ii) The given determinant is 3 5 −1
1
= [ −2( 4 − k)] 0 1 2
2
=k − 4 By definition of minors and co-factors, we have
∴ k − 4 = ±4 5 −1
M11 = Minor of a11 = = 10 + 1 = 11
When k − 4 = − 4, k = 0 1 2
When k − 4 = 4, k = 8 3 −1
M12 = Minor of a12 = =6−0=6
Hence, k = 0, 8 [2] 0 2
Q. 11. Write Minors and Co-factors of the elements of 3 5
following determinants : M13 = Minor of a13 = = 3−0 = 3
0 1
1 0 0
(i) 0 1 0 0 4
M21 = Minor of a21 = = 0 − 4 = −4
0 0 1 1 2
1 4
1 0 4 M22 = Minor of a22 = = 2−0= 2
(ii) 3 5 −1 0 2
 0 1 2 [NCERT Ex. 4.4, Q. 2, Page 126] 1 0
M23 = Minor of a23 = =1− 0 =1
1 0 0 0 1
Ans. (i) The given determinant is 0 1 0 0 4
M31 = Minor of a31 = = 0 − 20 = −20
0 0 1 5 −1
By the definition of minors and co-factors, we have 1 4
1 0 M32 = Minor of a32 = = −1 − 12 = −13
M11 = Minor of a11 = =1 3 −1
0 1 1 0
0 0 M33 = Minor of a33 = =5−0 =5
M12 = Minor of a12 = =0 3 5
0 1
A11 = Co-factor of a11 = (–1)1 + 1 M11 = 11
0 1 A12 = Co-factor of a12 = (–1)1 + 2 M12 = –6
M13 = Minor of a13 = =0
0 0 A13 = Co-factor of a13 = (–1)1 + 3 M13 = 3
A21 = Co-factor of a21 = (–1)2 + 1 M21 = 4
0 0
M21 = Minor of a21 = =0 A22 = Co-factor of a22 = (–1)2 + 2 M22 = 2
0 1 A23 = Co-factor of a23 = (–1)2 + 3 M23 = –1
1 0 A31 = Co-factor of a31 = (–1)3 + 1 M31 = –20
M22 = Minor of a22 = =1
0 1 A32 = Co-factor of a32 = (–1)3 + 2 M32 = 13
A33 = Co-factor of a33 = (–1)3 + 3 M33 = 5 [5]
1 0
M23 = Minor of a23 = =0 Q. 12. Using Co-factors of elements of third column,
0 0
1 x yz
0 0 evaluate ∆ = 1 y zx
M31 = Minor of a31 = =0
1 0 1 z xy
1 0  [NCERT Ex. 4.4, Q. 4, Page 126]
M32 = Minor of a32 = =0
0 0 1 x yz
1 0 Ans. The given determinant is ∆ = 1 y zx
M33 = Minor of a33 = =1
0 1 1 z xy
We have
A11 = Co-factor of a11 = ( −1 ) M11 = 1
1+1
1 y
M13 = =z−y
A12 = Co-factor of a12 = ( −1 )
1+ 2
M12 = 0 1 z
A13 = Co-factor of a13 = ( 1 ) M13 = 0 1 x
1+ 3
M23 = =z−x
1 z
A21 = Co-factor of a21 = ( −1 )
2 +1
M21 = 0
136 | OSWAAL NCERT SOLUTIONS – Textbook + Exemplar – MATHEMATICS : Class-XII

1 x 11 0 0 
M33 = = y−x
1 y = 0 11 0 
Therefore, 0 0 11
A13 = Co-factor of a13 = (–1)1 + 3 M13 = (z – y) Also,
A23 = Co-factor of a23 = (–1)2 + 3 M23 = – (z – x) = 0 3 2  1 −1 2 
(x – z)
∴ ( adj. A ) A =  −11 1 8  3 0 −2 
A33 = Co-factor of a33 = (–1)3 + 3 M33 = (y – x)
0 −1 3  1 0 3 
We know that Δ is equal to the sum of the product
of the elements of the second row with their 0 + 9 + 2 0+0+0 0−6+6 
corresponding co-factors.  
=  −11 + 3 + 8 11 + 0 + 0 −22 − 2 + 24 
Therefore, Δ = a13A13 + a23A23 + a33A33 0 − 3 + 3 0+0+0 0+2+9 
= yz ( z – y ) + zx ( x – z ) + xy ( y – x )
11 0 0 
= yz 2 − y 2 z + x 2 z + x 2 z − xz 2 + xy 2 – x 2 y  
= 0 11 0 
( ) ( ) (
= x 2 z – y 2 z + yz 2 – xz 2 + xy 2 – x 2 y ) 0 0 11
(
=z x –y 2 2
) + z ( y – x ) + xy ( y – x )
2
Hence, A ( adj. A ) = ( adj. A ) A = A I .
[5]
= z ( x – y )( x + y ) + z 2 ( y – x ) + xy ( y – x )
 2 −1 1
= ( x – y )  zx + zy – z 2 – xy  Q. 14. If A =  −1 
2 −1  Verify that A3 – 6A2 + 9A – 4I

= ( x – y )  z ( x – z ) + y ( z – x )   1 −1 2 
= ( x – y )( z – x ) [ − z + y ] =0 and hence find A–1.
= ( x – y )( y – z )( z – x )  [NCERT Ex. 4.5, Q. 16, Page 132]
Ans.
Hence, ∆ = ( x – y )( y – z )( z – x )
[5]  2 −1 1
Q. 13. Verify A ( adj. A ) = ( adj. A ) A = A I . A =  −1 2 −1 
 1 −1 2   1 −1 2 
 
 3 0 −2   2 −1 1   2 −1 1
 1 0 3     
 [NCERT Ex. 4.5, Q. 4, Page 131] A =  −1
2
2 −1   −1 2 −1 
Ans. Given that,  1 −1 2   1 −1 2 
1 −1 2  4 + 1 + 1 −2 − 2 − 1 2 + 1 + 2 
A = 3 0 −2  
=  −2 − 2 − 1 1 + 4 + 1 −1 − 2 − 2 
1 0 3   2 + 1 + 2 −1 − 2 − 2 1 + 1 + 4 
A = 1( 0 − 0) + 1(9 + 2) + 2( 0 − 0) = 11  6 −5 5 
 
1 0 0  11 0 0  =  −5 6 −5
∴ A I = 11 0 1 0  = 0 11 0   5 −5 6 
0 0 1  0 0 11  6 −5 5   2 −1 1
Now,   2 −1 
A 3 = A 2 A =  −5 6 −5  −1
A11 = 0, A12 = −(9 + 2) = −11, A13 = 0  5 −5 6   1 −1 2 
A21 = −( −3 − 0) = 3, A22 = 3 − 2 = 1, A23 = −( 0 + 1) = −1 12 + 5 + 5 −6 − 10 − 5 6 + 5 + 10 
A31 = 2 − 0 = 2, A32 = −( −2 − 6) = 8, A33 = 0 + 3 = 3 − − − −5 − 6 − 10 
=  10 6 5 5 + 12 + 5
0 3 2 10 + 5 + 6 −5 − 10 − 6 5 + 5 + 12 
∴ adj. A =  −11 1 8 

 22 −21 21 
0 −1 3   
=  −21 22 −21
Now,  21 −21 22 
1 −1 2  0 3 2 Now, A3 – 6A2 + 9A – 4I =0
  
A( adj. A ) = 3 0 −2   −11 1 8   22 −21 21   6 −5 5   2 −1 1 1 0 0
1 0 3  0 −1 3  =  −21 22 −21 − 6  −5 6 −5 + 9  −1 2 −1  − 4 0 1 0 
 21 −21 22   5 −5 6   1 −1 2  0 0 1 
0 + 11 + 0 3 − 1 − 2 2 − 8 + 6 
   22 −21 21   36 −30 30  18 −9 9   4 0 0
= 0 + 0 + 0 9 + 0 + 2 6 + 0 − 6 
=  −21 22 −21 −  −30 36 −30  +  −9 18 −9  − 0
 
4 0
0 + 0 + 0 3 + 0 − 3 2 + 0 + 9 
 21 −21 22   30 −30 36   9 −9 18  0 0 4 
DETERMINANTS | 137

 40 −30 30   40 −30 30  A21=8, A22=–10, A23=–6


A31=1, A32=3, A33=–5
=  −30 40 −30  −  −30 40 −30 
13 8 1
 30 −30 40   30 −30 40  −1 1 1  

= A =
( adj .A) 5 –10 3 
0 0 0 A 34 
3 – 6 –5
= 0 0 0 
1 
0 0 0  13 8 1  
1   3
∴ X= A −1B = 5 –10 3   
Now, we will find the value of A −1 34  2
3 – 6 –5  
A 3 – 6 A 2 + 9 A – 4I =
0 9 
⇒ ( AAA ) A −1 − 6( AA ) A −1 + 9 AA −1 − 4IA −1 =
0 x  13 + 12 + 9   34 
  1   1 
[Post-multiplying by A −1 as| A |≠ 0] ⇒
=  y  34 5 – 15 = + 27  17 
34 
 z  3 – 9 – 45   –51
⇒ AA( AA −1 ) − 6 A( AA −1 ) + 9 AA −1 =
4( IA −1 )
4 A −1
⇒ AAI − 6 AI + 9I =  
 1
2
⇒ A − 6 A + 9I = 4A −1
 
1
1 = 
⇒ A −1= ( A 2 − 6 A + 9I ) ...(i )  2
4  3
– 
A2 − 6 A + 9I  2
1 3
 6 −5 5   2 −1 1 1 0 0 Hence,=
x 1=
,y and z = –
2 2
= −5 6 −5 − 6  −1 2 −1  + 9 0 1 0 [5]
Q. 16. Solve system of linear equations, using matrix
 5 −5 6   1 −1 2  0 0 1 
method.
 6 −5 5   12 −6 6  9 0 0 x−y+z=4
=  −5 6 −5 −  −6 12 −6  + 0 9 0  2x + y − 3z = 0
 5 −5 6   6 −6 12  0 0 9  x+y+z=2
 [NCERT Ex. 4.6, Q. 12, Page 136]
 3 1 −1
Ans. The given system of equations can be written in the
=  1 3 1 form of AX = B, where
 –1 1 3 1 −1 1  x  4
From equation (i), we have =A  2 1 −3 , =
X  y  and =
B 0 
 3 1 −1  1 1 1   z   2 
1
A −1 =  1 3 1  Now,
4
 −1 1 3 A =1(1 + 3) + 1( 2 + 3) + 1( 2 − 1) =4 + 5 + 1 =10 ≠ 0
[5]
Q. 15. Solve system of linear equations, using matrix Thus, A is non-singular. Therefore, its inverse
method. exists.
2x + y + z =1
Now, A11=4, A12=–5, A13=1
3 A21=2, A22=0, A23=–2
x − 2y − z =
2 A31=2, A32=5, A33=3
3y − 5 z =
9
 [NCERT Ex. 4.6, Q. 11, Page 136] 4 2 2
Ans. The given system of equations can be written in the −1 1 1  
∴ A =( adj. A ) = −5 0 5 
form of AX = B, where A 10
1 −2 3 
1 
2 1 1  x  3 4 2 2 4
    1 
A = 1 −2 −1 , X =  y  and B=   ∴ X= A −1B = −5 0 5  0 
 
2 10
0 3 −5  z  9  1 −2 3   2 
 
Now, x  16 + 0 + 4   20 
⇒  = 1  −20 + 0 + 10 = 1  −10 
|A|=2(10+3) –1(–5–3)+0  y  10   10  
    =2(13) –1(–8)  z   4 + 0 + 6  10 
    =26+8 2 
    =34≠0 =  −1
Thus, A is non-singular. Therefore, its inverse
1 
exists.
Now, A11=13, A12=5, A13=3 2, y =
Hence, x = −1 and z =
1
[5]
138 | OSWAAL NCERT SOLUTIONS – Textbook + Exemplar – MATHEMATICS : Class-XII

Q. 17. Solve system of linear equations, using matrix 7 1 −3


method. 1 1
∴ A −1 = ( adj. A ) =  −19 −1 11 
2x + 3y + 3z = 5 A 4
  x − 2y + z = −4  −11 −1 7 
 3x − y − 2z = 3 7 1 −3 7 
 [NCERT Ex. 4.6, Q. 13, Page 136] 1  
∴ X = A −1B =  −19 −1 11   −5
Ans. The given system of equations can be written in the 4
 −11 −1 7  12 
form of AX = B, where
2 3 3  x  5  x   49 − 5 − 36  8 
  = 1  −133 + 5 + 132  = 1  4 
A = 1 −2 1  , X =  y  and B =  −4  ⇒ y
  4  4 
3 −1 −2   z  3   z   −77 + 5 + 84  12 

Now, 2
A = 2( 4 + 1) − 3( −2 − 3) + 3( −1 + 6) = 1 
3 
= 2(5) − 3( −5) + 3(5)
= 10 + 15 + 15 Hence, x = 2, y = 1 and z = 3
[5]
= 40 ≠ 0  2 −3 5 
 
Thus, A is non-singular. Therefore, its inverse Q. 19. If A =  3 2 −4  find A–1. Using A–1 solve
exists.  1 1 −2 
Now, A11=5, A12=5, A13=5  
A21=3, A22=–13, A23=11 the system of equations.
A31=9, A32=1, A33=–7 2 x − 3 y + 5 z = 11
3 x + 2 y − 4 z = −5
5 3 9 
1 1 
−1
∴ A = ( adj. A ) = 5 −13 1  x + y − 2 z = −3
A 40   [NCERT Ex. 4.6, Q. 15, Page 137]
5 11 −7 
5 3 9  5  Ans. Given that,  2 −3 5 
1   
  A =  3 2 −4 
∴ X = A −1B = 5 −13 1   −4 
40  1 1 −2 
5 11 −7  3   
x   25 − 12 + 27   40  ∴ A = 2 ( −4 + 4 ) + 3 ( −6 + 4 ) + 5 ( 3 − 2 )

⇒ y =  1  
25 + 52 + 3  =
1 
80  = 0 − 6 + 5 = −1 ≠ 0
40  40 
 z   25 − 44 − 21  −40  Now,
A11 = 0, A12 =2, A13 = 1
1 
  A21 = –1, A22 = –9, A23 = –5
=2 
  A31= 2, A32 = 23, A33 =13
 −1
0 −1 2  0 1 −2 
Hence, x = 1, y = 2 and z = −1 1    
[5] −1
∴A = ( adj.A ) = −  2 −9 23 =  −2 9 −23
Q. 18. Solve system of linear equations, using matrix A 1 −5 13   −1 5 −13
method.    
  x − y + 2z = 7 ...(i)
3x + 4y − 5z = −5 Now, the given system of equations can be written
    2x − y + 3z = 12 in the form of AX = B, where
 [NCERT Ex. 4.6, Q. 14, Page 136]  2 −3 5  x  11 
Ans. The given system of equations can be written in the  2     
A = 3 −4  , X =  y  and B=  −5
form of AX = B, where 1 1 −2   −3
 z 
1 −1 2  x  7     
A = 3 4 −5 , X =  y  and B =  −5
   
The solution of the system of equations is given
 2 −1 3   z  12  by X = A–1B
Now, X = A −1B

A = 1(12 − 5) + 1(9 + 10) + 2( −3 − 8)  x  0 1 −2  11 


    
= 7 + 19 − 22 ⇒  y  =  −2 9 −23  −5 [By using Eq. (i)]
  
=4≠0  z   −1 5 −13  −3
Thus, A is non-singular. Therefore, its inverse exists. 0 − 5 + 6 
Now, A11=7, A12=–19, A13=–11  
=  −22 − 45 + 69 
A21=1, A22=–1, A23=–1  −11 − 25 + 39 
A31=–3, A32=11, A33=7  
DETERMINANTS | 139
1  Q. 21. If a, b and c are real numbers, and
  b+ c c+ a a+ b
= 2
3  ∆= c + a a + b b + c = 0 .Show that either a +
  a+ b b+ c c+ a
Hence, x =1, y = 2 and z = 3 [5]
Q. 20. The cost of 4 kg onion, 3 kg wheat and 2 kg rice b + c = 0 or a = b = c.
is Rs. 60. The cost of 2 kg onion, 4 kg wheat and 6  [NCERT Misc. Ex. Q. 4, Page 141]
kg rice is Rs. 90. The cost of 6 kg onion 2 kg wheat Ans. Given that,
and 3 kg rice is Rs. 70. Find cost of each item per b+c c+a a+b
kg by matrix method.
 [NCERT Ex. 4.6, Q. 16, Page 138] ∆= c + a a + b b + c
Ans. Let the cost of onions, wheat and rice per kg be a+b b+c c+a
` x, ` y and ` z respectively. Then the given situation
Applying R1 → R1 + R2 + R3
can be represented as :
4x + 3y + 2 z = 60 2( a + b + c ) 2( a + b + c ) 2( a + b + c )
2x + 4 y + 6 z =90 ∆= c + a a+b b+c
6 x + 2 y + 3z =70 a+b b+c c+a
This system of equations can be written in the form
1 1 1
of AX = B, where
4 3 2 60  = 2( a + b + c ) c + a a + b b + c
x 
      a+b b+c c+a
= A 2 = 4 6  , X = y  and B 90 
6 2 3   z  70  Applying C2 → C2 − C1 and C3 → C3 − C1 , we have:
   
A = 4(12 − 12) − 3( 6 − 36) + 2( 4 − 24) 1 0 0
=0 + 90 − 40 =50 ≠ 0 ∆= 2( a + b + c ) c + a b − c b − a
Now, A 11 = 0, A12 = 30, A13 = −20 a+b c−a c−b
−5, A22 =
A21 = 0, A23 =10 Expanding along R1 , we have :
10, A32 =
A31 = −20, A33 =10 ∆ = 2 ( a + b + c ) (1) [( b − c )( c − b ) − (b − a)( c − a)]
0 −5 10  = 2( a + b + c )  − b 2 − c 2 + 2bc − bc + ba + ac − a 2 
 
∴ adj= . A 30 0 −20  = 2( a + b + c )  ab + bc + ca − a 2 − b 2 − c 2 
 −20 10 10 
  It is given that ∆ =0.
0 −5 10  ( a + b + c )  ab + bc + ca − a 2 − b 2 − c 2  = 0
1 1  
∴ A= −1
adj. A= 30 0 −20  ⇒ Either a + b + c= 0, ab + bc + ca − a 2 − b 2 − c 2= 0.
A 50 
−20 10 10 
  Now,
Now,
X = A–1B ab + bc + ca − a 2 − b 2 − c 2 =
0
0 −5 10  60  ⇒ −2ab − 2bc − 2ca + 2a 2 + 2b 2 + 2c 2 =
0
1    ⇒ ( a − b )2 + ( b − c )2 + ( c − a )2 =
0
=⇒ X 30 0 −20  90 
50 
−20 10 10  70  ⇒ ( a − b )2 = ( b − c )2 = ( c − a )2 = 0
  
[( a − b )2 ,( b − c )2 ,( c − a )2 are non-negative]
x   0 − 450 + 700 
  1   ⇒ (a − b ) = (b − c ) = ( c − a ) = 0

=  y  50 1800 + 0 − 1400  ⇒a=b=c
 z   −1200 + 900 + 700 
  Hence, if Δ = 0, then either a + b + c = 0 or a =
 250  b = c.[5]
1  
=  400  x+a x x
50 
400  Q. 22. Solve the equation x x+a x =
0, a ≠ 0
 
x x x+a
5 
   [NCERT Misc. Ex. Q. 5, Page 141]
= 8 
8  Ans. Given that,
  x+a x x
∴ Hence, x = 5, y =8 and z = 8
x x+a x =0
Hence, the cost of onions is ` 5 per kg, the cost of
wheat is ` 8 per kg, and the cost of rice is ` 8 per kg. x x x+a
 [5]
140 | OSWAAL NCERT SOLUTIONS – Textbook + Exemplar – MATHEMATICS : Class-XII

Applying R1 → R1 + R2 + R3 , we get
a c a+c
3x + a 3x + a 3x + a = 2ab c a + b b a
2

x x+a x =0 1 1 0
x x x+a
Applying C2 → C2 − C1 , we have:
1 1 1 a c−a a+c
⇒ (3x + a) x x + a x =0 ∆ = 2ab c a + b − a
2
a
x x x+a 1 0 0
Applying C2 → C2 − C1 and C3 → C3 − C1 , we have:
Expanding along R3, we have
1 0 0 ∆ = 2 ab 2 c [ a( c − a) + a( a + c )]
( 3x + a ) x a 0 = 0 = 2 ab 2 c[ ac − a 2 + a 2 + ac ]
x 0 a
= 2 ab 2 c [2 ac ]
Expanding along R1 , we have
= 4a2 b 2 c 2
( 3x + a ) 1 × a2  = 0 Hence, the given result is proved. [5]
⇒ a 2 (3x + a) = 0 Q. 24. If
But a ≠ 0 3 −1 1  1 2 −2 
   
0  , find ( AB ) .
−1
Therefore,we have : −1
A =  −15 6 −5 and B =  −1 3
3x + a = 0 5 −2 2   0 −2 1 
   
a
⇒ x=−  [NCERT Misc. Ex. Q. 7, Page 141]
[5] 3
Ans. We know that ( AB)−1 = B −1 A −1
a2 bc ac + c 2 1 2 −2 
 
Q. 23. Prove that a + ab b
2 2
ac = 4 a 2 b2 c 2 B =  −1 3 0 
ab b + bc c
2 2 0 −2 1 
 
 [NCERT Misc. Ex. Q. 6, Page 141] ∴ B = 1 × (3 − 0) − 2(–1) – 2(2 – 0) = 3 + 2 − 4 = 5 − 4 = 1
Ans. Let we assume that,
Now, A11 = 3, A12 = 1, A13 = 2
a2 bc ac + c 2 A21 = 2 , A22 = 1, A23 = 2
∆ = a 2 + ab b 2 ac A31 = 6, A32 = 2 , A33 = 5
ab b 2 + bc c 2
3 2 6 
Taking out common factors a , b , c from C1 , C2 and  
∴ adj.B = 1 1 2 
C3 , we have 2 2 5 
 
a c a+c
Now,
∆ = abc a + b b a
1
b b+c c B −1 = .adj.B
|B|
Applying R2 → R2 − R1 and R3 → R3 − R1 we have 3 2 6 
a+c  
a c ∴ B−1 = 1. 1 1 2 
∆ = abc b b − c −c  2 2 5 
b−a b −a ∴ ( AB ) = B−1 A −1
−1

Applying R2 → R2 + R1 , we have 3 2 6  3 −1 1 
 
a c a+c = 1 1 2   −15 6 −5
∆ = abc a + b b a  2 2 5  5 −2 2 
b − a b −a 9 − 30 + 30 −3 + 12 − 12 3 − 10 + 12 

Applying R3 → R3 + R2 , we have = 3 − 15 + 10 −1 + 6 − 4 1 − 5 + 4 
6 − 30 + 25 −2 + 12 − 10 2 − 10 + 10 
a c a+c
9 −3 5 
∆ = abc a + b b a − 
= 2 1 0
2b 2b 0
1 0 2 
[5]
Now,
 −14 −11 5
1 
A −1 = −11 −4 3
13  DETERMINANTS | 141
5 3 1 
1 −2 1   14 11 5 
Q. 25. Let A =  −2 3   − 13 −
13 13 
 1 Verify that  
11 4 3
 1 1 5 = − −
 13 13 13 
–1
(i)  adj. A =adj. A–1 ( ) 
5 3 1

13 13 13 
(A )
–1
(ii) =A
–1
 4 9  11 15  33 20 
 [NCERT Misc. Ex. Q. 8, Page 142]  − 169 − 169 − − −  − + 
Ans. Given that,   169 169  169 169 
  11 15  14 25  42 55  
1 −2 1 ∴ adj. A −1  −  − −  − − − − +
 169 169  

A =  −2 3    169 169  169 169  
1
 33 20  42 55  56 121 
1 1 5 − + − − +  − 
 169 169  169 169  169 169 
∴ A = 1(15 − 1) + 2( −10 − 1) + 1( −2 − 3)
 −13 26 −13
=14 − 22 − 5 1  
= 26 −39 −13
= − 13 169 
 −13 −13 −65
Now,
A11 = 14, A12 = 11, A13 = –5  −1 2 −1
1  
A21 = 11, A22 = 4, A23 = –3 = 2 −3 −1
13 
A31 = –5, A32 = –3, A13 = –1   −1 −1 −5
[2]
14 11 −5
Hence, [ adj. A]−1 = adj.( A −1 ).
∴ adj. A = 11 4 −3 (ii)
(A )
–1
–1
=A
 −5 −3 −1
We have shown that,
1
∴ A −1 = ( adj.A ) 1
 −14 −11 5
A 
A −1 =  −11 −4 3
14 11 −5 13
5 3 1 
1 
=− 11 4 −3
13   −1 2 −1
 −5 −3 −1 1
and, adj. A =  2 −3 −1
−1

 −14 −11 5 13
 −1 −1 −5
=  −11 −4 3
1 
13 Now,
5 3 1  [1] 3
1
(i) [ adj.A ]
–1
= adj. A ( ) –1 A −1 =   [ −14( −13) + 11( −26) + 5( −13)]
 13 
3
adj. A = 14( −4 − 9) − 11( −11 − 15) − 5( −33 + 20) 1
=   ( −169)
= 14( −13) − 11( −26) − (5)( −13)  13 
1
= −182 + 286 + 65 =−
     13
= 169
We have, adj. A −1
(A )
−1
−1
∴ =
 −13 26 −13 A −1
adj.( adj. A ) =  26 −39 −13  −1 2 −1
1 1 
 −13 −13 −65 = × 2 −3 −1
 1  13 
∴ [ adj. A]−1 =
1
( adj.( adj.A ))  − 13   −1 −1 −5
adj. A  
1 −2 1 
 −13 26 −13
1  =  −2 3 1 
= 26 −39 −13
169  1 1 5
 −13 −13 −65
=A
 −1 2 −1
( )
−1
1   Hence, A −1 =A
=  2 −3 −1 [2]
13
 −1 −1 −5 x y x+y
Now,
Q. 26. Evaluate y x+y x
 −14 −11 5
1  x+y x
−11 −4 3
y
A −1 =
13   [NCERT Misc. Ex. Q. 9, Page 142]
5 3 1 
Ans. Let we assume that,
 14 11 5 
 − 13 − 13 13 
 
11 4 3
= − −
142 | OSWAAL NCERT SOLUTIONS – Textbook + Exemplar – MATHEMATICS : Class-XII

x y x+y Q. 28. Using properties of determinants, prove that :


3a −a + b −a + c
=∆ y x+y x
− b + a 3b − b + c= 3( a + b + c )( ab + bc + ca)
x+y x y
− c + a − c + b 3 c
Applying R1→R1+R2+R3, we have
 [NCERT Misc. Ex. Q. 13, Page 142]
2( x + y ) 2( x + y ) 2( x + y )
Ans. Taking LHS, we get
=∆ y x+y x 3a −a + b −a + c
x+y x y ∆ = −b + a 3b −b + c
1 1 1 −c + a −c + b 3c
=
2( x + y ) y x+y x Applying C1 → C1 + C2 + C3 , we have
x+y x y a + b + c −a + b −a + c
Applying C2→C2–C1 and C3→C3–C1, we have ∆= a + b + c 3b −b + c
1 0 0 a + b + c −c + b 3c
=
∆ 2( x + y ) y x x−y
1 −a + b −a + c
x + y − y −x
=( a + b + c ) 1 3b −b + c
Expanding along R1, we get
1 − c + b 3c
∆ 2( x + y )  − x 2 + y( x − y )
=
Applying R2 → R2 − R1 and R3 → R3 − R1 , we have
=
−2( x + y )( x 2 + y 2 − yx ) 1 −a + b −a + c
= −2( x 3 + y 3 ) [5] ∆= ( a + b + c ) 0 2b + a a − b
Q. 27. Using properties of determinants, prove that :
0 a−c 2c + a
x x 2 1 + px 3
Expanding along C1, we have
y y 2 1 + py 3 = (1 + pxyz )( x − y )( y − z )( z − x ) ∆= ( a + b + c )[( 2b + a )( 2c + a ) − ( a − b )( a − c )]
2 3
z z 1 + pz , = ( a + b + c )[ 4bc + 2ab + 2ac + a 2 − a 2 + ac + ba − bc]
where p is any scalar
 [NCERT Misc. Ex. Q. 12, Page 142] = ( a + b + c )(3ab + 3bc + 3ca )
Ans. Let we assume that, = 3( a + b + c )( ab + bc + ca )
x x 2 1 + px 3 Hence proved. [5]
Q. 29. Solve the system of the following equations
=∆ y y 2 1 + py 3 2 3 10
z z 2 1 + pz 3 + + = 4
x y z
Applying R2 → R2 − R1 and R3 → R3 − R1 , we have 4 6 5
− + = 1
x x 1 + px
2 3 x y z
∆= y − x y − x p( y 3 − x 3 )
2 2 6 9 20
+ − =
2
z − x z 2 − x 2 p( z 3 − x 3 ) x y z [NCERT Misc. Ex. Q. 16, Page 142]
Ans. Given that,
x x2 1 + px 2 2 3 10
= ( y − x )( z − x ) 1 y + x p( y 2 + x 2 + xy ) + + = 4
x y z
1 z + x p( z 2 + x 2 + xz ) 4 6 5
− + = 1
Applying R3 → R3 − R2 , we have x y z
x x2 1 + px 3 6 9 20
+ − = 2
∆= ( y − x )( z − x ) 1 y + x p( y 2 + x 2 + xy ) x y z
0 z − y p( z − y )( x + y + z ) Then the given system of equations is as follows
2 p + 3q + 10r = 4
x x2 1 + px 3
4 p − 6q + 5r = 1
=( y − x )( z − x )( z − y ) 1 y + x p( y 2 + x 2 + xy )
6 p + 9q − 20r = 2
0 1 p( x + y + z )  
Expanding along R3, we have This system can be written in the form of AX = B,
where
∆= ( x − y )( y − z )( z − x )
 2 3 10  p 4
( −1)( p )( xy + x + x y ) + 1 + px + p( x + y + z )( xy )
2 3 2 3
A=  4 −6 5  , X = q  and B =
1 
     
= ( x − y )( y − z )( z − x ) 6 9 −20  r   2 
 − pxy 2 − px 3 − px 2 y + 1 + px 3 + px 2 y + pxy 2 + pxyz  Now, A 2 (120 – 45 ) – 3 ( –80 – 30 ) + 10 (36 + 36 )
=
=( x − y )( y − z )( z − x )[1 + pxyz] = 150 + 330 + 720
Hence proved. [5] = 1200
DETERMINANTS | 143
Thus, A is non-singular. Therefore, its inverse ⇒ (cos A − cos C )(cos B − cos C )
exists.
0 0 1
Now,
1 1 1 + cos C =
0
A11=75, A12=110, A13=72
cos A + cos C + 1 cos B + cos C + 1 cos2 C + cos C
A21=150, A22=–100, A23=0
[Applying C1 → C1 − C2 ]
A31=75, A32=30, A33=–24
⇒ (cos A − cos C )(cos B − cos C )
1 −1
∴ A = adj. A 0 0 1
A
0 1 1 + cos C =
0
75 150 75 
=
1 
110 −100 30 
 cos A − cos B cos B + cos C + 1 cos2 C + cos C
1200  Expanding along C1 , we get
72 0 −24 
Now, ⇒ (cos A − cos C )(cos B − cos C )(cos B − cos A ) =
0
X = A −1B ⇒=cos A cos C or= cos B cos C or= cos B cos A
p 75 150 75   4  ⇒=A C or= B C or= B A
  1    Hence, ∆ABC is an isosceles triangle.
⇒= q
  1200  110 −100 30  1 
r  72 0 −24   2  [5]
300 + 150 + 150  0 1 1 
=
1  
440 − 100 + 60  Q. 31. Find A−1 if A =  1 0 1  and show that
 
1200   1 1 0 
 288 + 0 − 48  2
A − 3I
600  A −1 =
1  2
= 400   [NCERT Exemp. Ex. 4.3, Q.17, Page 79]
1200 
 240  0 1 1 
 
1 Ans. Given that, A = 1 0 1 
2 1 1 0 
 
1
=  Co-factors are:
3 
  A11 = −1, A12 = 1, A13 =1
1 
 5  A21 = 1, A22 = −1, A23 = 1
1 1 1 A31 = 1, A32 = 1, A33 = −1
∴ p = , q = and r = T
2 3 5  −1 1 1   −1 1 1 
  = 
Hence,=
x 2= , y 3 and= z 5
[5] ∴ adj. A = 1 −1 1  1 −1 1 
Q. 30. Show that the ΔABC is an isosceles triangle if the 1 1 −1 1 1 −1
determinant A = 0 − 1( −1) + 1.1 = 2
1 1 1  −1 1 1 
adj. A 1 
∆ = 1 + cos A 1 + cos B 1 + cos C = 0 ∴ −1
A = = 1 −1 1 
A 2
2 2 2
cos A + cos A cos B + cos B cos C + cos C 1 1 −1
[NCERT Exemp. Ex. 4.3, Q.16, Page 78, Now,
CBSE Board, All India Region, 2016] 0 1 1  0 1 1   2 1 1 
A 2 = 1 0 1  1 0 1  = 1 2 1 
 
Ans. We have,
1 1 0  1 1 0  1 1 2 
1 1 1
∆ = 1 + cos A 1 + cos B 1 + cos C = 0
1 1 1   2 1 1  3 0 0  
+ Bcos A 1 cos
1 + cos A cos 1 +Acos
2
+ cos CB + cos
2
= 0B cos C + cos C
2
A 2 − 3I 1   
=  1 2 1  − 0 3 0  
∆=
cos2 A + cos A cos2 B + cos B cos2 C + cos C

[Applying C1 → C1 − C3 and C2 → C2 − C3 ] 2 2 
[Applying C1 → C1 − C3 and C2 → C2 − C3 ]
0 0 1
 1 1 2  0 0 3  
0 0 1
⇒ cos A −⇒ cos A − cos C cos B − cos C
cos C cos B − 1cos CC
+ cos = 0 1 + cos C =0−1 1 1 
1
cos2 A + coscos
A −2cos − cosA
A +C cos cosB2 +CcosB
C −cos − cos − cos2CB +cos
C C cos C +−
cos C 2 C − cos C C 1 −1 1  =
2 2 2 2
− cos cosB cos cos2 C + cos=
 A −1
2
[Taking (cosA–cosC) common from C1 and (cosB– 1 1 −1
cosC) common from C2] Hence proved. [5]
144 | OSWAAL NCERT SOLUTIONS – Textbook + Exemplar – MATHEMATICS : Class-XII

2 2 −4   1 −1 0 
1 2 0    and  
  Q. 33. Given A =  − 4 2 − 4  B = 2 3 4  find
Q. 32. If A =−  2 − 1 −2 
,  2 −1 5   0 1 2 
 0 −1 1 
BA and use this to solve the system of equations
Using A–1, solve the system of linear equations y +2z = 7, x – y = 3 and 2x + 3y + 4z = 17.
x– 2y=10, 2x – y – z = 8 and – 2y + z = 7.  [NCERT Exemp. Ex. 4.3, Q.20, Page 79]
 [NCERT Exemp. Ex. 4.3, Q.18, Page 79] Ans. We have,
Ans. We have,
 2 2 −4  1 −1 0 
1 2 0   −4 2 −4  and B = 2 3 4
  A=    
A =− 2 −1 −2  ...(i)
 2 −1 5  0 1 2 
0 −1 1 
1 −1 0   2 2 −4   6 0 0
∴ A =1( −3) − 2( −2) + 0 =1 ≠ 0
=∴ BA  2 3 4   −4 2 = −4  0 = 
 6 0 6I
Now,
0 1 2   2 −1 5  0 0 6
A11 =
−3, A12 =
2, A13 =
2
 2 2 − 4
A21 =
−2, A22 =
1, A23 =
1 A 1
∴ B = =  −4 2 −4 
−1
....(i)
A31 =
−4, A32 =
2, A33 =
3 6 6
T  2 −1 5 
 −3 2 2   −3 −2 −4  Given system of equations is
∴ adj. A =−  =  
 2 1 1 2 1 2  y +2z = 7,
 −4 2 3   2 1 3  x–y=3
and 2x + 3y + 4z = 17
 −3 −2 −4 
or
adj. A 1  
∴ A −1= = . 2 1 2  ∴ AX = B
A 1
 2 1 3  1 −1 0   x  3 
 −3 −2 − 4      
−1    2 3 4  y  = 17 
⇒ A = 2 1 2  .....(ii) 0 1 2  z  7 
 2 1 3  ∴X = −1
A B
Also, we have the system of linear equation as −1
       x–2y=10,  x  1 −1 0  3   2 2 −4 3 
  2x – y – z = 8       1  
= ∴  y   2 3 4 17  =  −4 2 −4 17 
and – 2y + z = 7 6
 z  0 1 2 7   2 −1 5  7 
In the form of CX=D,
1 −2 0   x  10  6 + 34 − 28  12 
 2 −1 −1  y  = 8  1  1 
=  −12 + 34 − 28 =  −6
     6 6
0 −2 1   z  7  6 − 17 + 35   24 

1 −2 0  x  10  2 
       
where, C=  2 −1 −= =
1 , X  y  and D 8  =  −1
0 −2 1   z  7   4 
We know that, Hence, x = 2, y = −1 and z = 4
[5]
T −1 −1 T
(A ) = ( A ) a b c
1 2 0  Q. 34. If a + b + c ≠ 0 and b c a = 0, then prove that
T   a = b = c. c a b
∴ C = −2 −1 −2  =A [By using Eq. (i)]
0 −1 1   [NCERT Exemp. Ex. 4.3, Q. 21, Page 79]
a b c
∴ X=C −1D Ans. Given that, a + b + c ≠ 0 and b c a = 0,
 x   −3 2 2  10  c a b
⇒  y  =  −2 1 1  8 
   a b c
 z   −4 2 3  7  Let A= b c a
 −30 + 16 + 14 
c a b
=  −20 + 8 + 7 
a+b+c a+b+c a+b+c
 −40 + 16 + 21 = b c a [ R1 → R1 + R2 + R3 ]
0  c a b
 
=  −5 1 1 1
 −3
= (a + b + c) b c a
Hence, x = 0, y = −5 and z = −3 [5] c a b
1
⇒ − = −1
b
⇒ b =1 DETERMINANTS | 145
and
0 0 1 −3 − a
= ( a + b + c ) b − a c − a a [ C1 → C1 − C3 and C 2 → C 2 − C3 ] ⇒ =1
b
c−b a−b b ⇒ −3 − a = 1
Expanding along R1 , ⇒ a = −4
= ( a + b + c )[1( b − a )( a − b ) − ( c − a )( c − b )]
Hence, given determinant is divisible by (a + b + c)
= ( a + b + c )( ba − b 2 − a 2 + ab − c 2 + cb + ac − ab ) and quotient is
1
= − ( a + b + c ) × ( −2)( − a 2 − b 2 − c 2 + ab + bc + ca ) ( a3 + b 3 + c 3 − 3abc )[( a − b )2 + ( b − c 2 ) + ( c − a )2 ] [5]
2
Q. 36. If x +y +z =0, then prove that
1
= − ( a + b + c )[ a 2 + b 2 + c 2 − 2ab − 2bc − 2ca + a 2 + b 2 + c 2 ] xa yb zc a b c
2
1 yc za xb = xyz c a b
= − ( a + b + c )[ a 2 + b 2 − 2ab + b 2 + c 2 − 2bc + c 2 + a 2 − 2ca]
2 zb xc ya b c a
1
= − ( a + b + c )[( a − b )2 + ( b − c )2 + ( c − a )2 ]  [NCERT Exemp. Ex. 4.3, Q. 23, Page 80]
2
Also, A = 0 Ans. Given that, x +y +z =0
1 xa yb zc a b c
⇒ − ( a + b + c )[( a − b )2 + ( b − c )2 + ( c − a )2 ] = 0
2 Now, yc za xb xyz c a b
⇒ ( a − b )2 + ( b − c )2 + ( c − a )2 = 0 [ a + b + c ≠ 0, given] zb xc ya b c a
⇒ a−b =b−c = c−a =0 Taking LHS, we get
⇒a=b=c xa yb zc
[5]
Hence proved. LHS = yc za xb
bc − a2 ca − b2 ab − c 2 zb xc ya
Q. 35. Prove that ca − b2 ab − c 2 bc − a2 is divisible
= xa( za.ya − xb.xc ) − yb( yc.ya − xb.zb ) + zc( yc.xc − za.zb )
ab − c 2 bc − a2 ca − b2
= xa( a 2 yz − x 2bc ) − yb( y 2 ac − b 2 xz ) + zc( c 2 xy − z 2 ab )
by (a + b + c) and find the quotient.
= xyza3 − x 3abc − y 3abc + b 3xyz + c 3xyz − z 3abc
 [NCERT Exemp. Ex. 4.3, Q. 22, Page 79]
bc − a 2 ca − b 2 ab − c 2 = xyz( a3 + b 3 + c 3 ) − abc( x 3 + y 3 + z 3 )
Ans. Given that, ca − b 2 ab − c 2 bc − a 2 =xyz( a3 + b 3 + c 3 ) − abc(3xyz )
ab − c 2 bc − a 2 ca − b 2 [ x + y + z = 0 ⇒ x 3 + y 3 + z 3 − 3xz]
Let we assume that, = xyz( a3 + b 3 + c 3 − 3abc ) .....(i)

bc − a 2 ca − b 2 ab − c 2 a b c
∆ = ca − b 2 ab − c 2 bc − a 2 Now, RHS = xyz c a b
ab − c 2 bc − a 2 ca − b 2 b c a

bc − a 2 − ca + b 2 ca − b 2 − ab + c 2 ab − c 2 a+b+c b c
= ca − b 2 − ab + c 2 ab − c 2 − bc + a 2 bc − a 2 = xyz a + b + c a b [ C1 → C1 + C2 + C3 ]
ab − c 2 − bc + a 2 bc − a 2 − ca + b 2 ca − b 2 a+b+c c a

[ C1 → C1 − C2 and C2 → C2 − C3 ] 1 b c
= xyz( a + b + c ) 1 a b
( b − a )( a + b + c ) ( c − b )( a + b + c ) ab − c 2
1 c a
= ( c − a )( a + b + c ) ( a − c )( a + b + c ) bc − a 2
[Taking ( a + b + c ) common from C1 ]
( a − c )( a + b + c ) ( b − a )( a + b + c ) ca − b 2
0 b−c c−a
b − a c − b ab − c 2
= xyz( a + b + c ) 0 a − c b − a
= ( a + b + c ) c − a a − c bc − a 2
2
1 c a
a − c b − a ca − b 2
[ R1 → R1 − R3 and R2 → R2 − R3 ]
[T aking (a + b + c ) common from C1 and C2 each]
Expanding along C1 ,
0 0 ab + bc + ca − ( a 2 + b 2 + c 2 )
= xyz( a + b + c )[1( b − c )( b − a ) − ( a − c )( c − a )]
= ( a + b + c )2 c − b a − c bc − a 2
= xyz( a + b + c )( b 2 − ab − bc + ca + a 2 + c 2 − 2ac )
a − c b − a ca − b 2
= xyz( a + b + c )( a 2 + b 2 + c 2 − ab − bc − ca )
[ R1 → R1 + R2 + R3 ]
= xyz( a + b + c )( a 2 + b 2 + c 2 − ab − bc − ca )
1
⇒ − = −1 = xyz( a + b + c )( a 2 + b 2 + c 2 − ab − bc − ca )
b
⇒ b =1 = xyz( a3 + b 3 + c 3 − 3abc ) ...(ii)
and From equations (i) and (ii), we get
−3 − a LHS=RHS
⇒ =1
b xa yb zc a b c
⇒ −3 − a = 1 ⇒ yc za xb = xyz c a b
146 | OSWAAL NCERT SOLUTIONS – Textbook + Exemplar – MATHEMATICS : Class-XII

LHS=RHS Therefore, we have AB = 67 × 61 − 87 × 47 =


xa yb zc a b c 4087 − 4089 = −2
⇒ yc za xb = xyz c a b Also,
zb xc ya b c a 61 −87 
[5] adj.( AB) =  
Hence proved.  − 47 67 
3 7  6 8  1 1 61 −87 
∴ ( AB ) =
−1
Q. 37. Let A =   and B =   . Verify that .adj.( AB) = −  
2 5 7 9  AB 2 −47 67 
( AB ) = B−1 A−1
−1
 61 87 
 [NCERT Ex. 4.5, Q. 12, Page 132] − 2 
= 2  ....(ii)
3 7   47 67
Ans. We have, A =   − 
2 5   2 2 
From (i) and (ii), we have
A = 15 − 14 = 1
(AB)–1 = B–1A–1.
Now, Hence, the given result is proved. [5]
A11 = 5, A2 = −2 3 1 
A21 = −7 , A22 = 3 Q. 38. If A =   show that A2 − 5 A + 7 I = 0 . Hence
 −1 2 
5 −7  find A–1.
∴ adj. A =  
 −2 3   [NCERT Ex. 4.5, Q. 13, Page 132]
1 5 −7  Ans. Given that,
∴ A −1 = .adj. A = 1.   3 1 
A  −2 3  A= 
 −1 2 
6 8 
Now, let B =   3 1  3 1 
7 9  A 2 = A. A =   
 −1 2   −1 2 
B = 54 − 56 = −2 9 − 1 3+ 2 
=
Now,  −3 − 2 −1 + 4 
B11 = 9 , B12 = −8 8 5
B21 = −7 , B22 = 6 = 
 −5 3
9 −8  ∴ A 2 − 5 A + 7I
∴ adj.B =  
 − 7 6  8 5 3 1  1 0 
 9  =  − 5  −1 2  + 7 0 1 
− 4   −5 3     
−1 1 1 9 −8   2
∴B = .adj.B = − .   = 
A 2  −7 6   7 8 5 15 5  7 0 
−3 = − +
 2   −5 3  −5 10  0 7 
Now,
 −7 0  7 0 
 9  = +
 − 2 4  5 −7  0 −7  0 7 
−1 −1
B A =  
 7 − 2 3 0 0 
−3   = 
 2  0 0 
 45 63  Hence, A 2 − 5 A + 7 I = 0
 − 2 − 8 2 + 12 
=  ∴ A. A − 5 A = −7 I
 35 + 6 49
− − 9 ⇒ A. A( A −1 ) − 5 A. A −1 = −7 IA −1
 2 2 
[Post-multiplying by A −1 as A ≠ 0]
 61 87 
− 2 2  ⇒ A( AA −1 ) − 5I = −7 A −1
=  ...(i)
 47 67 ⇒ AI − 5I = −7 A −1
− 
 2 2  1
Then, ⇒ A −1 = − ( A − 5I )
7
3 7   6 8  1
AB=    ⇒ A −1 = ( 5I − A )
 2 5  7 9  7
18 + 49 24 + 63 1  5 0  3 1  
=  ⇒ A −1 =   − 
12 + 35 16 + 45  7  0 5   −1 2  
67 87  1  2 −1
=  ⇒ A −1 =  
 47 61 7 1 3 
[5]
DETERMINANTS | 147
3 2 Ans.
Q. 39. For the matrix A =   find the numbers a and
1 1 −1
 
1 1  A = 1 2 −3
b such that A2 +aA + bI = 0.  2 −1 3 
 [NCERT Ex. 4.5, Q. 14, Page 132]  
Ans. 1 1 1  1 1 1 
3 2    
A= A 2 = 1 2 −3 1 2 −3
  2 −1 3   2 −1
1 1     3 

3 2  3 2 
∴ A 2 = A. A =    1 + 1 + 2 1 + 2 − 1 1 − 3 + 3  4 2 1 
1 1  1 1     
= 1 + 2 − 6 1 + 4 + 3 1 − 6 − 9  =  −3 8 −14 
9 + 2 6 + 2  2 − 1 + 6 2 − 2 − 3
=  2 + 3 + 9  7 −3 14 
3 + 1 2 + 1     
11 8  4 2 1  1 1 1 
=    
 4 3 A = A . A =  −3 8
3 2
−14  1 2 −3
Now, 7 −3 14   2 −1 3 
   
A 2 + aA + bI = 0 4 + 2 + 2 
4 + 4 −1 4−6+3
⇒ ( AA ) A −1 + aAA −1 + bIA −1 = 0  
=  −3 + 8 − 28 −3 + 16 + 14 −3 − 24 − 42 
[Post-multiplying by A −1 as A ≠ 0] 7 − 3 + 28 7 − 6 − 14 7 + 9 + 42 
 
( ) (
⇒ A AA −1 + aI + b IA −1 = 0) 8 7 1 
−1
⇒ AI + aI + bA = 0  
=  −23 27 −69 
⇒ A + aI = −bA −1 32 −13 58 
 
1
⇒ A −1 = − ( A + aI ) ∴ A 3 − 6 A 2 + 5 A + 11I
b
Now, 8 7 1  4 2 1 
   
1 1 1 −2  1 −2  =  −23 27 −69 − 6  −3 8 −14
A −1 = adj. A =  =  32 −13 58  7 −3 144 
A 1  −1 3   −1 3     
We have,
1 1 1  1 0 0
1 −2  1  3 2   a 0      
 −1 3  = −  1 1  + 0 a   +5 1 2 −3 + 11 0 1 0
  b     2 −1 3  0 0 1 
   
1 3 + a 2 
=−  8 1   24 12 6 
b 1 1 + a  
7
  
=  −23 27 −69 −  −18 48 −84
 −3 − a 2 
 b − 32 −13 58   42 −18 84 
= b     

− 1 −1 − a  5 5 −5  11 0 0 
 b b     
+ 5 10 −15 + 0 11 0 
Comparing the corresponding elements of the two 10
matrices, we have −5 15  0 0 11
   
1  24 12 6   6   0 0 0 
⇒ − = −1 24 12
b      
=  −18 48 −84 −  −18 48 −84 = 0 0 0 = 0
⇒ b =1  42 −18 84   42 −18 84  0 0 0
and      
−3 − a Thus, A3 – 6A2 +5A +11I =0
⇒ =1
b Now,
⇒ −3 − a = 1 A3 – 6A2 +5A +11I =0
⇒ a = −4 ⇒ ( AAA ) A −1 − 6( AA ) A −1 + 5 AA −1 + 11IA −1 = 0
Hence, −4 and 1 are the required values of a and b [Post-multiplying by A −1 as A ≠ 0]
respectively. [5]
⇒ AA( AA −1 ) − 6 A( AA −1 ) + 5( AA −1 ) = −11( IA −1 )
1 1 1 
  ⇒ A 2 − 6 A + 5I = −11A −1
Q. 40. For the matrix A =  1 2 −3  . Show that
1
 2 −1 3 
 
⇒ A −1 = −
11
(
A 2 − 6 A + 5I ) ...(i)

A3– 6A2 +5A +11I= 0. Hence, find A–1. Now,


 [NCERT Ex. 4.5, Q. 15, Page 132] A2 – 6A+5I
148 | OSWAAL NCERT SOLUTIONS – Textbook + Exemplar – MATHEMATICS : Class-XII

4 2 1  1 1 1  1 0 0 0 −1 2  0 1 −2 
         
=  −3 8 −14  − 6 1 2 −3 + 5 0 1 0 ∴ A −1 = −1  2 −9 23 =  −2 9 −23
7 −3 14  2 −1 3  0 0 1 1 −5 13   −1 5 −13
         
4 2 1  6 6 6  5 0 0  We can write the given equation as AX = B
     
=  −3 8 −14  − 6 12 −18 + 0 5 0  2 −3 5   x  11 
7 −3 14  12 −6 18  0 0 5     
      3 2 −4   y  =  −5
1 1 −2   z   −3
9 2 1   6 6 6  5 0 0     
     
=  −3 13 −14  − 6 12 −18 + 0 5 0   x  0 1 −2  11 

7 −3 19  12 −6 18  0 0 5      
⇒  y  =  −2 9 −23  −5
     
 z   −1 5 −13  −3
9    
2 1  6 6 6 
     x  1 
=  −3 13 −14  − 6 12 −18
7    
−3 19  12 −6 18  ⇒  y  = 2
     z  3 
3   
−4 −5
  ∴ x = 1, y = 2 and z=3
=  −9 1 4  [6]
 −5 3 1   1 −1 0  2 
  2 −4
   
Q. 42. If A =  2 3 4  and B =  −4 2 −4  are two
From equation (i), we have 0 1 2  2 −1 5 
3 −4 −5  −3 4 5   
1  1  square matrices, find AB and hence solve the
A −1 = −  −9 1 4  = 9 −1 −4 
11  11 system of linear equations x – y = 3; 2x + 3y + 4z
−5 3 1  5 −3 −1 
= 17 and y + 2z = 7.
    [5]
 [CBSE Board, Foreign Scheme, 2017]
 2 −3 5  Ans.
  –1
Q. 41. If A =  3 2 −4  find A Use it to solve the 6 0 0 
1 1 −2   
  Getting AB = 0 6 0  = 6I
system of equations. 0 0 6 
2 x − 3 y + 5 z = 11  
3 x + 2 y − 4 z = −5 Given system of equations can be written as
x + y − 2 z = −3 1 −1 0   x  13 
    
 [CBSE Board, Delhi Region, 2018]  2 3 4   y  = 17 
Ans. Given that, 0 1 2   z  7 
   
 2 −3 5  1  1 
  i.e., AX = C ⇒ X = A −1C = BC  AB = 6I ⇒ A −1 = B 
A = 3 2 −4  6  6 
1 1 −2 
   2 2 −4  3   2 
1    
⇒ A = 2( −4 + 4) + 3( −6 + 4) + 5(3 − 2) =  −4 2 −4  17  =  −1
6    
= −6 + 5  2 −1 5  7   4 
= −1
⇒ x = 2, y = −1, z = 4 [6]
Now,
T yz − x 2
zx − y 2
xy − z 2

 −4 
( −1)
1 +1 2
( −1)1+ 2 3 −4 ( −1)1+ 2 3 2  Q. 43. Prove that zx − y 2 xy − z 2 yz − x 2 is divisible
 1 −2 1 −2 1 1  xy − z 2 yz − x 2 zx − y 2
 
adj. A = ( −1)2 +1 1 5 ( −1)2 + 2 2 5 ( −1)2 + 3 2 −3  by (x + y + z), and hence find the quotient.
 −3 −2 1 −2 1 1 
  Ans. Using C1 → C1 – C3 and C2→ C2 – C3 we get
 3 +1 −3 
( −1)
5 ( −1)3+ 2 2 5 ( −1)3+ 3 2 −3  Taking (x +y+ z) common from C1 and C2
 2 −4 3 −4 3 2 
y( z − x ) + z 2 − x 2 x( z − y ) + z 2 − y 2 xy − z 2
0 2 1 
T
0 −1 2  ∆ = z( x − y ) + x 2 − y 2 y( x − z ) + x 2 − z 2 yz − x 2
    x( y − z ) + y 2 − z 2 z( y − x ) + y 2 − x 2 zx − y 2
=  −1 −9 −5 =  2 −9 23
 2 23 13  1 −5 13  Taking (x + y + z) common from C1 and C2
   
DETERMINANTS | 149
2 Q. 45. A shopkeeper has 3 varieties of pens ‘A’, ‘B’
z − x z − y xy − z
and ‘C’. Meenu purchased 1 pen of each variety
⇒ ∆= ( x + y + z )2 x − y x − z yz − x 2
for a total of Rs. 21. Jeevan purchased 4 pens of
y − z y − x zx − y 2 ‘A’ variety, 3 pens of ‘B’ variety and 2 pens of ‘C’
R1 → R1 + R2 + R3 variety for Rs. 60. While Shikha purchased 6 pens
of ‘A’ variety, 2 pens of ‘B’ variety and 3 pens of ‘C’
0 0 xy + yz + zx − x 2 − y 2 − z 2
variety for Rs. 70. Using matrix method, finfd the
⇒ ∆= ( x + y + z ) x − y x − z yz − x 2 cost of each variety of pen.
y − z y − z zx − y 2  [CBSE Board, All India Region, 2016]
Ans. Let the cost of one pen of variety ‘A’, ‘B’ and ‘C’
Expanding to get
be ` x, ` y and ` z respectively then the system of
∆ = (x + y + z)2 ( xy + yz +zx – x2– y2 –z2)2
equations is :
Hence ∆ is divisible by (x+ y+ z) and the quotient x+y+z= 21
is (x + y + z) ( xy + yz +zx – x2– y2 –z2)2 [6]
4x + 3y + 2 z = 60
Q. 44. Using elementary transformations, find the
8 4 3 6x + 2 y + +3z =70
  Matrix form of the system is :
inverse of the matrix A =  2 1 1  and use it to
 1 2 2  1 1 1  x   21
       
solve the following system of linear equations : = A.X B= , where A  4 3 2 = ; X=  y  ; B 60 
8x + 4y + 3z = 19 6 2 3   z  70 
   
2x + y + z = 5
x + 2y + 2z = 7 A =(5) − 1( 0) + 1( −10) =−5
 [CBSE Board, Delhi Region, 2016] Co-factors of the matrix A are :
8 4 3  C11 =5; C21 = –1; C31 = –1
  C12 = 0; C22 = –3; C32 = –1
Ans. Given that, A =  2 1 1 
1 2 2  C13 = –10 C23 = 4; C33 = –1
  5
8
−1 −1
4 3  1 0 0  1 1  
    ∴= A −1
=
Adj .A 0 −3 2 
2 1 1  = 0 1 0 A A −5 
1 2 2  0 0 1  −10 4 −1
     
1 2 2 0 0 1  Solution of the matrix equation is X = A–1 B
    5
R1 ↔ R3 2 1 1 = 0 1 0 A x  −1 −1   21 5
8    1    
 4 3  1 0 0
 ⇒  y  =− 0 −3 2 − 1 60  =8  ∴ x =5, y =8, z =8
   5  70  8 
 z  −10 4
 −3 0 0  0 −2 1       [6]
   
R1 → R1 − 2R2 and R3 → R3 − 4R2  2 1 1  =0 1 0 A 1+ a 1 1
    Q. 46. If a, b and c are all non-zero and 1 1+ b 1 =
0 0 −1 1 −4 0  0
1 1 1+ c
1 0 0  0 2 / 3 −1 / 3 1 1 1
1     then prove that + + = 0
R1 → R1 and R3 → − R3  2 1 1  =0 1 2 /3 A a b c
3 0 0 1  1 4 
  
0

 [CBSE Board, Foreign Scheme, 2016]
1 0 0  0 2 / 3 −1 / 3 1+ a 1 1
    Ans. Given that,     1
R2 → R2 − 2R1 0 1 1  = 0 −1 / 3 2 / 3  A 1+ b 1 0
=
0 0 1   −1 4 0  1 1 1+ c
   
1 0 0  0 2 / 3 −1 / 3 1 1 1
    +1
−1 / 3 2 / 3  A a b c
R2 → R2 − R3 0 1 0  = 1
0 0 1   −1  1 1 1
4 0 abc +1 0
=
    a b c
0 2 / 3 −1 / 3  1 1 1
  +1
A −1 = 1 −13 / 3 2 / 3            a b c
 −1 4 0  C1 → C1 + C2 + C3
 
 1 1 1 1 1 
AX =B ⇒ X = A −1B 1 + a + b + c b c 
 
0 2 / 3 −1 / 3 19  1  1 1 1 1 1 
x 
     ⇒ abc 1 + + + +1 0
=
   a b c b c 
=∴ y 1 = −13 / 3 2 / 3  5  2  
 z   −1 4  7  1  1 + 1 + 1 + 1 1 1 
+1
0
       a b c b c 
∴ x= 1, y= 2, z= 1 [6]
150 | OSWAAL NCERT SOLUTIONS – Textbook + Exemplar – MATHEMATICS : Class-XII

 1 1   cos α − sin α 0 
1 b c   
  A =  sin α cos α 0
1 1  0
 1 1 1
 +1 0 1 
⇒ abc 1 + + +  1 b c =0 
 a b c  1 1 

 [CBSE Board, Foreign, 2016]
 +1
1 b c  Ans.
  A =1
 
R2 → R2 − R1 , R3 → R3 − R1 cosa sin a 0
 
 1 1 1 adj. A =  − sin a cosa 0
⇒ abc 1 + + +  = 0 0
 a b c 0 1
 
 a, b, c, ≠ 0 1 0 0 
1 1 1  
∴ 1+ + + = 0 A ( adj. A ) = 0 1 0  = I
a b c [6] 0 0 1 
Q. 47. Find adj.A and verify that A(adj.A) = (adj.A)A =  
|A|I3. If 1 0 0 
 
A I 3 = 0 1 0  = I
0 0 1 
  [6]

Some Commonly Made Errors


 Students do not write the formula when doing numerical.
 Students confuse in Inverse and Transpose of the matrix.
 Students do not put right sign convention when finding adjoint.
 Students get confuse in finding co-factor and minor of the matrix.

EXPERT ADVICE
☞ For the sums of Matrix Elementary Operation, do not change rows and columns together in the same sum.
☞ In the Determinant sums, to get full marks, students must use the properties of determinant.
☞ For row transformation, change only row, and for column transformation, change only column.
☞ Always Try to Use Direct Methods for the Solution of Linear Algebraic Equations.

OSWAAL LEARNING TOOLS


For Suggested Online Videos
Visit : https://goo.gl/hiQo5F Visit : https://goo.gl/Cpikvd

Or Scan the Code Or Scan the Code

  

You might also like